Download as pdf or txt
Download as pdf or txt
You are on page 1of 79

Philippine Association of Colleges of Pharmacy

PUBLICH HEALTH, IMMUNOLOGY;

1. The following are found in the cell wall of certain bacteria. Which of the following is exclusively found among
fungi?
A. Teichoic acid
B. Diaminopimelic acid
C. Peptidoglycan
D. Chitin
2. The endotoxin of gram-negative bacteria is associated with
A. Mesosome
B. Cytoplasmic membrane
C. Capsule
D. Cell wall
3. This is a specific staining method for spirochetes
A. Giemsa’s stain
B. Ziehl-neilsen stain
C. Acid-fast stain
D. Gram’s stain
4. Eukaryotes differ from prokaryotes in that eukaryotes
A. Generally lack sterols in their cell membrane
B. Contain more than one chromosome
C. Lack 80’s ribosome
D. Have peptidoglycan
5. These are straight filaments arising from the bacterial cell wall, making the bacterium look like porcupine
A. Capsules
B. Endospores
C. Fimbriae
D. Flagella
6. The tolerance of facultative anaerobic bacteria to lethal superoxide when they are exposed to air is due to
A. The lack of CYP C oxidase
B. Presence of CYP C oxidase
C. Inability to form the superoxide radical
D. Presence of superoxide dismutase and catalase
7. Lipopolysaccharides are correctly described as
I. Found in both Gram(-) and Gram(+) bacteria
II. Contains lipid A as part of its structure
III. Infection of which are treated by antitoxin
A. I only
B. II only
C. I and II only
D. II and III only
E. I, II, and III
8. A structural component of the cell that may enhance virulence of bacterium include
A. Capsule
B. Hyaluronidase
C. Flagella
D. Lecithinase
9. The source of H antigen
A. Flagella
B. Cell wall
C. Pilus
D. Plasmid
10. The process in which DNA is released by lysis of one bacterium, leading to a change in phenotype of that
another bacterium is called
A. Transduction
B. Transformation
C. Conjugation
D. Transposition

Microbiology, Public Health and Immunology


Philippine Association of Colleges of Pharmacy

11. Transfer of genetic materials involving the so called “jumping genes”


A. Transduction
B. Transformation
C. Conjugation
D. Transposition
12. A process of gene transfer which involves bacteriophages in the transfer of DNA material from one bacterium to
another
A. Transduction
B. Transformation
C. Conjugation
D. Transposition
13. Transfer of genetic materials which involves sex pilus
A. Transduction
B. Transformation
C. Conjugation
D. Transposition
14. Microbial mutations are due to which of the following cause(s)
I. Inversion
II. Additions (insertions)
III. Substitutions (transversions)
A. I only
B. II only
C. I and II only
D. II and III only
E. I, II and III
15. Endospores have a multi-layered protective coat consisting of
I. Cell membrane
II. Peptidoglycan mesh
III. Exosporium
A. I only
B. II only
C. I and II only
D. II and III only
E. I, II and III
16. Which of the following statement(s) about plasmids is/are true
I. Carry resistance genes for antibiotics called R plasmids
II. Don’t control genes regulating enzymes capable of destroying antimicrobial drugs
III. Can’t be transferred by conjugation due to their size and shape
A. I only
B. II only
C. I and II only
D. II and III only
E. I, II and III
17. The most likely cells to be lysed when treated with anti-CD4 antibody and complement are
A. Cytotoxic t cells
B. Macrophages
C. Helper T cells
D. B cells
18. Cytotoxic T cells are best characterized by the following cell-surface marker
A. CD4
B. CD8
C. CD3
D. CD2
19. The major function of interferon-gamma is to
A. Induce the activation of resident macrophages
B. Inhibit migration of macrophages from reaction sites
C. Promote and maintain the proliferation of T cells
D. Promote B and T cell growth
20. Cytotoxic T cells are involved in which of the following action(s)
I. Antigen presentation
II. Tumor destruction
III. Cell lysis of virus infected cells
A. I only
Microbiology, Public Health and Immunology
Philippine Association of Colleges of Pharmacy

B. II only
C. I and II only
D. II and III only
E. I, II and III
21. T-helper cells are important in augmentation of which of the following
I. Antibody production
II. Mixed lymphocyte reaction
III. Delayed-type hypersensitivity reactions
A. I only
B. II only
C. I and II only
D. II and III only
E. I, II and III
22. Which of the following statement(s) is/are correct for Natural-Killer cells
I. Recognize and destroy certain tumor cells
II. Lyse virus-infected cells
III. Involve in delayed-type hypersensitivity reactions
A. I only
B. II only
C. I and II only
D. II and III only
E. I, II and III
23. A male patient suffering from bacterial infection received a penicillin injection and he almost immediately had
respiratory distress. This reaction is most likely to be mediated by
A. T cells
B. IgE
C. IgG and IgM
D. IgG and Complement
24. AIDS patients infected with TB are likely to react less vigorously to tuberculin test because of the deficiency of
A. Platelets
B. CD8
C. CD4
D. B cells
25. An AIDS patient who received a tuberculin test 48 hours after showed a weak erythematous skin reaction (6mm
in diameter) at the site of injection. What is the proper interpretation for the observation? The patient,
A. Has never been exposed to TB bacilli
B. Has active TB
C. Has been exposed to TB bacilli
D. Is infected with multidrug-resistant TB bacilli
26. The ability of the T cells to discriminate between self and non-self is learned in the
A. Liver
B. Spleen
C. Lymph nodes
D. Thymus
27. Which of the following will grow in the presence of optochin
I. Streptococcus pyogenes
II. Streptococcus viridans
III. Streptococcus pneumonia
A. I only
B. II only
C. III only
D. I and III only
E. II and III only
28. Which statement is true for passive immunization
I. Provides long lasting protection
II. Provides protection without hypersensitivity
III. Provides immediate protection
A. I only
B. II only
C. III only
D. I and III only
E. II and III only

Microbiology, Public Health and Immunology


Philippine Association of Colleges of Pharmacy

29. A vaccine antigen that primarily induces opsonizing antibody is


A. Hepatitis B virus vaccine
B. MMR vaccine
C. Rabies vaccine
D. H. influenza Type B vaccine
30. Streptococci are incubated in a blood agar plate overnight. Which of the following reactions show beta-hemolytic
streptocci?
A. Completely lyse the RBC
B. Partially lyse the RBC
C. Unable to hemolyse the RBC
D. Leave a greenish discoloration
31. The following contribute(s) to the development of a compromised state of the host defense
I. Radiation therapy
II. Exposure to toxoid
III. Malnutrition
A. I only
B. I and II only
C. II and III only
D. I and IIII only
E. I, II and III only
32. Which of the following statement(s) is/are correct
I. Toxoides are used as immunogen
II. Exotoxins are highly toxic at very low doses
III. Some G(-) bacteria exotoxins
IV. Some G(+) bacteria produce endotoxins
A. I only
B. I and II only
C. II and III only
D. I and IV only
E. I, II and III only
33. Which of the following is not a correct statement concerning neutrophils
A. They can be differentiated into B cells on activation
B. They constitute the first line of nonspecific host defense mechanism
C. Principal phagocytic cells together with monocytes and macrophages
D. They cannot present antigens
34. Diagnostic lab techniques most frequently employed for the visual microscopic detection of antigens in tissue
sections, in cells suspensions, or on cell monolayers include
A. Agglutination
B. Radioimmunoassay
C. Immunofluorescence
D. Precipitation
35. This is/are forces or bonds involved in the interaction between an antigen and an antibody
I. Covalent bonds
II. Van der waals
III. H-bonds
A. I only
B. I and II only
C. II and III only
D. I, II and III
36. A man with blood group phenotype AB
A. May donate blood to individuals with blood type O
B. Has anti-A and anti-B in his serum
C. Has A and B antigen in his RBC
D. May be given blood type AB, A, or B but not Type O
37. Subacute bacterial endocarditis (SBE) is caused by
A. Staphylococcus aureus
B. Streptococcus pyogenes
C. Staphylococcus saprophyticus
D. Streptococcus viridans

Microbiology, Public Health and Immunology


Philippine Association of Colleges of Pharmacy

38. Immunologic suppression for transplantation may be induced by


I. Lymphoid radiation
II. Cyclosporine
III. Steroids
A. I only
B. I and II only
C. III only
D. I and III only
E. I, II and III
39. The maturation of T-cells in the thymus is accompanied by changes in the
I. Cell surface markers
II. Cell size
III. Surface immunoglobulin class
A. I only
B. II only
C. III only
D. I and II
E. I and III
40. Which of the following organisms are correctly paired with their microbial organelles or components known to
enhance their virulence or invasiveness?
I. M protein of Strep. pyogenes
II. Pili of N. gonorrhoeae
III. Mesosome of Staph. aureus
A. I only
B. II only
C. I and II
D. I and III
E. I, II and III
41. Toxic activities of endotoxin reside in the
I. Muramic acid component of peptidoglycan
II. Lipid A moiety of lipopolysaccharide
III. Capsular polysaccharide
A. I only
B. II only
C. III only
D. I and II
E. I and III
42. A vaccine produced from an extracellular toxic bacterial product that is made nontoxic, but maintains
antigenicity, is known as
A. Live attenuated organism
B. Antitoxin
C. Toxoid
D. Killed organism
43. Which statement(s) describes a monocyte
I. They can become macrophages
II. They only populate the lymph nodes
III. They are actively phagocytic
A. I only
B. II only
C. III only
D. I and II
E. I and III
44. Which of the following statements best describes an antigen
I. Is always an immunogen
II. Has one or more epitopes
III. Molecular weight may be less than 10,000 are potently immunogenic
A. I only
B. II only
C. III only
D. I and II
E. I and III

Microbiology, Public Health and Immunology


Philippine Association of Colleges of Pharmacy

45. A small molecule that can react with preformed antibodies but cannot, by itself, induce their formation is referred
to as
A. Hapten
B. Epitope
C. Complete antigen
D. Cofactor
46. Opsonin-treated bacteria are more readily engulfed by phagocytes than are untreated bacteria because
I. The capsule is removed by opsonin
II. Opsonin digests wall component
III. The surgace of phagocytes contains receptors for opsonins
A. I only
B. II only
C. III only
D. I and II
E. I and III
47. The first line of defense against viruses is
A. Interferon
B. IgM directed to internal viral antigens
C. IgG directed to external viral antigens
D. All of the above
48. Type I hypersensitivity
A. Is also known as delayed-type hypersensitivity
B. Involves IgE response to an allergen
C. Is independent of inflammatory mediators
D. Involved degranulation of eosinophils
49. Which of the following diseases is caused by an exotoxin release of Staphylococcus aureus
A. Gastroenteritis
B. Osteomyelitis
C. Meningitis
D. Pneumonia
50. The maximal reaction time for tuberculin reaction is
A. 2-5 hours
B. 6-10 hours
C. 12-20 hours
D. 48-72 hours
51. Memory cells
A. Lymphocytes
B. Macrophages
C. Polymorphonuclear leukocytes
D. Basophils
52. First white blood cells to be involved in acute inflammation by pyogenic cocci
A. Macrophages
B. Polymorphonuclear leukocytes
C. Basophils
D. Lymphocytes
53. Antigen presenter
A. Dendritic macrophages
B. NK cells
C. Eosinophils
D. Pre-B cells
54. Helminth infections will cause an increase in
A. NK cells
B. Dendritic macrophages
C. Eosinophils
D. Pre-B cells
55. Vaccine for Rubeola
A. Toxoid
B. Toxin
C. Living attenuated microorganisms
D. Killed attenuated microorganisms

Microbiology, Public Health and Immunology


Philippine Association of Colleges of Pharmacy

56. Vaccine for Whooping cough


A. Toxoid
B. Toxin
C. Living attenuated microoganisms
D. Killed attenuated microogranisms
57. Vaccine for Tetanus
A. Toxoid
B. Toxin
C. Living attenuated microorganisms
D. Killed attenuated microorganisms
58. Autoimmune haemolytic anemia and Goodpasteur’s syndrome
A. Immediate hypersensitivity
B. Cytotoxic antibody
C. Immune-complex disease
D. Delayed-type hypersensitivity
E. NOTA
59. Asthma
A. Immediate hypersensitivity
B. Cytotoxic antibody
C. Immune-complex disease
D. Delayed-type hypersensitivity
E. NOTA
60. Contact dermatitis
A. Immediate hypersensitivity
B. Cytotoxic antibody
C. Immune-complex disease
D. Delayed-type hypersensitivity
E. NOTA
61. Tetanus, botulism and diphteria
A. Immediate hypersensitivity
B. Cytotoxic antibody
C. Immune-complex disease
D. Delayed-type hypersensitivity
E. NOTA
62. A burn patient developed a wound infection, and a bacteriologic culture of the site indicated a gram-negative rod
that was oxidase-positive and produced a blue-green pigment. The organism was relatively resistant to
antibiotics but susceptible to ticarcillin, gentamicin and tobramycin. This organism is likely
A. E. coli
B. K. pneumonia
C. S. marcescens
D. P. aeruginosa
63. A spinal fluid specimen from a 28 year old man shows N. meningitides. The DOC is
A. Pen G
B. Amikacin
C. Kanamycin
D. Ethambutol
64. Pending the results of cultures, meningitis in children 8-10 months of age should be treated with
A. Ampicillin
B. Pen G
C. Cefortaxime
D. Tetracycline
65. Tetracycline may be the DOC for all EXCEPT
A. LGV
B. Inclusion conjunctivitis
C. Hospital-acquired PNX due to Klebsiella
D. Psittacosis
66. Penicillin is the DOC for the following infections caused by EXCEPT
A. Treponema pallidum
B. Streptococcus pyogenes
C. Bacteriodes fragilis
D. Clostridium perfringens

Microbiology, Public Health and Immunology


Philippine Association of Colleges of Pharmacy

67. Which of the following is true of staphylococcus epidermidis


I. Catalase-positive
II. Coagulase-positive
III. Facultative
A. I only
B. II only
C. III only
D. I and II
E. I and III
68. An adult patient with a history of severe penicillin allergy has a sore throat and low-grade fever. Results of a
throat culture show a large number of beta-hemolytic streptococcal colonies. Of the following, which best suits
the patient?
A. Ampicillin
B. Cephalothin
C. Tetracycline
D. Phenoxymethylpenicillin
E. Erythromycin
69. All of the following are modified penicillins resistant to destruction by staphylococcal beta-lactamase except
A. Naficillin
B. Ampicillin
C. Oxacillin
D. Cloxacillin
E. Methicillin
70. Treatment of herpetic conjunctivitis
A. Streptomycin
B. Iododeoxyuridine
C. Rifampin
D. Nalidixic acid
71. Effective given orally in Microsporum-induced disease of hair
A. Metronidazole
B. Griseofulvin
C. Nystatin
D. Polymyxin
72. Streptomycin is known for
A. Erythema nodosum leprosum
B. Bone marrow depression
C. Discoloration of the teeth
D. Vestibular damage
73. Dapsone is given for
A. Erythema nodosum leprosum
B. Bone marrow depression
C. Discoloration of the teeth
D. Vestibular damage
74. Polypeptide antibiotic inhibiting pseudomonas
A. Amphotericin B
B. Colistin
C. Carbenicillin
D. Methicillin
75. Inhibits the mucopeptide synthesis by affecting alanine racemase
A. Amphotericin B
B. Colistin
C. Cycloserine
D. Methicillin
76. A healthy medical student clearing in Lyme, Connecticut, was bitten by ticks. One week later complains of fever,
stiff neck, stiff joints and arrhythmias. An expanding skin lesion known as erythema chronicum migrans was
found on her leg with sharply formed borders. DOC for this condition is
A. Metronidazole
B. Pen V
C. Flucytosine
D. Chloramphenicol

Microbiology, Public Health and Immunology


Philippine Association of Colleges of Pharmacy

77. A 22 year-old nurse who had been in excellent health complained of fatigue and diarrhea. Then he developed
pneumonia that was caused by Pneumocystis carinii. Blood tests repeated showed depletion of CD4 T-
lymphocutes. The patient must be managed with
A. 3’-azidothymidine
B. Chloroquine
C. Acyclovir
D. Enviroxime
78. Which of the following is an example of an aerobic organisms?
A. Listeria monocytogenes
B. Histoplasma capsulatum
C. Mycobacterium tuberculosis
D. Staphylococcus aureus
79. The toxin of E. coli that is very similar to the shiga toxin of Shigella dysenteriae is
A. Heat-labile enterotoxin
B. Heat-stable enterotoxin
C. Verotoxin
D. Exfoliative toxin
80. The most recently discovered sexually transmitted disease is
A. Trichomoniasis
B. Condyloma acuminate (warts)
C. Hepatitis
D. AIDS
81. Congenital syphilis can be best detected by the use of
A. Dark-field examination
B. Silver nitrate staining of the spirochetes
C. The Wassermann Complement fixation test
D. IgM-FTA-ABS test
82. Humans acquire Salmonella typhimurium by
A. Ingestion of contaminated food and water
B. Tick bites
C. Mosquito bites
D. Aerosols
83. The main virulence factor of Yersinia pestis is/are the
A. Endotoxin
B. V and W antigens
C. Erythrogenic toxins
D. Lecithinase
84. Mycoplasma are pleomorphic gram-negative bacteria that
A. Are sensitive to penicillin
B. Lack cell walls in all stages of growth
C. Cannot divide by binary fission
D. Do not produce “fried egg” colonies
85. The spirochete that is tightly coiled, thin, and has a hook at its end is
A. Leptospira
B. Borrelia
C. Treponema
D. Vibrio
86. The body louse is the bector for
A. Leptospirosis
B. Pinta
C. Epidemic relapsing fever
D. Meningitis
87. Chlamydia trachomatis infection may result to
A. Brill’s disease
B. Ornithosis
C. Undulant fever
D. Lymphogranuloma venerum
88. In the laboratory diagnosis of the enteric pathogens Salmonella and Shigella, the screening procedure includes
testing their inability to
A. Ferment glucose
B. Ferment lactose
C. Reduce nitrates
D. Produce oxidase
Microbiology, Public Health and Immunology
Philippine Association of Colleges of Pharmacy

89. Characterized with marked hypotension, and scarlatiniform rash followed by desquamation
A. Toxic shock syndrome
B. Botulism
C. Cholera
D. Tetanus
90. Causative agent of undulant fever
A. Brucella
B. Listeria
C. Legionella
D. Chlamydia
91. A nonmotile, oxidase-positive, gram-negative coccus, usually in pairs, that is a facultative anaerobe with urethral
epithelium in the male
A. Gonococcus
B. Meningococcus
C. Pneumococcus
D. Staphylococcus
92. All of the following are transmitted to man by ingestion of undercooked meat. Which of the following is vector
transmitted?
A. Trichinella
B. Leishmania
C. Clonorchis
D. Diphyllobothrium
93. A major cause of blindness in tropical Africa is
A. Ancylostoma duodenale
B. Enterobius vermicularis
C. Trichinella spiralis
D. Onchocerca volvulus
94. A cestode that competes with its host for dietary Vit. B12
A. Hymenolepsis nana
B. Taenia solium
C. Taenia saginata
D. Diphyllobothrium latum
95. Vector of Trypanosoma cruzi
A. Sand Fly (Phlebotomus)
B. Tsetse Fly (Glossina)
C. Reduviid (Kissing Bug)
D. Black Fly (Simulium)
96. The event that correlates with the presence of fever paroxysm in Plasmodium vivax malaria is
A. Invasion of hepatocytes by sporozoites
B. Invasion of new RBC by merozoites
C. Schizont rupture
D. Gametocytes
97. Natural resistance to malaria caused by Plasmodium vivax has been demonstrated in populations who are
A. Duffy blood group antigen-positive
B. G6PD deficient
C. Duffy blood group antigen-negative
D. All of the above
98. Flagellated protozoa that cause gastroenteritis in humans include
A. Entamoeba histolytca
B. Trichomonas vaginalis
C. Balantidium coli
D. Giardia lamblia
99. Generally infects human eating raw beef
A. Taenia solium
B. Taenia saginata
C. Trichinella spiralis
D. Necator americanus
100. This adult parasites are usually in the parenchyma of the human lungs
A. Paragonimus westermanii
B. Clonorchis sinensis
C. Schistosoma japonicum
D. Taenia solium

Microbiology, Public Health and Immunology


Philippine Association of Colleges of Pharmacy

101. A 5 year old girl spent a year attending schoolon West Africa. She returnes to the US because of poor
sleeping and intense itching in her anal area. Anal swab samples of the patient would most likely contain the
larvae of which organism?
A. Wuchereria bancorft
B. Trichinella spiralis
C. Enterobius vermicularis
D. Schistosoma japponicum
102. The Filovidae are a newly recognized family of negative-sense, single-stranded RNA viruses. Which of the
following viruses belongs to this family and causes hemorrhagic fever?
A. Marburg virus
B. Yellow fever virus
C. Dengue virus
D. Parvovirus
103. Corona virus are recognized by club-shaped surface projections that are 20 nm long and resemble solar
coronas. These viruses are characterized by their ability to
A. Infect infants more frequently than adults
B. Cause common colds
C. Grow well in the usual cultures cell line
D. Agglutinate human red blood cells
104. Mumps virus is biologically relates to the virus causing which of the following disease?
A. Rabies
B. Variola
C. Vaccine
D. Measles
105. Which of the following antiviral agent is purine nucleoside analogue that has shown promise with lassa
fever, influenza A and B and respiratory syncytial virus (RSV)?
A. Amantadine
B. Ribavirin
C. Acyclovir
D. Gancyclovir
106. ECHOviruses are cytopahogenic human viruses that mainly infects the
A. Respiratory tract
B. Central Nervous System
C. Blood and lymphatic system
D. Intestinal tract
107. A 5 year old child presents with symptoms of low-grade fever, coryza, conjunctivitis and Koplik’s spots. The
causative agent of this disease belongs to which group of viruses?
A. Adenovirus
B. Paramyxovirus
C. Piconavirus
D. Herpesvirus
108. Human rotavirus is best described by which of the following statements
A. It is often associated with infantile diarrhea
B. It I an example of viral STD
C. It is associated with conjunctivitis
D. All of the above
109. Varicella is a member of which of the following viral families
A. Pox virus
B. Adenovirus
C. Herpes virus
D. Paramyxovirus
110. German measles virus is a member of which viral family
A. Herpes
B. Pox virus
C. Togavirus
D. Orthomyxovirus
111. A patient has all the gastrointestinal symptoms of infection with Hepatitis A virus (HAV), yet all tests for HAV-
IgG and HAV-IgM are non-reactive A possible cause of this infection id
A. Hepa B surface antigen
B. Hepa C surface antigen
C. Hepa D surface antigen
D. Hepa E surface antigen

Microbiology, Public Health and Immunology


Philippine Association of Colleges of Pharmacy

112. The human papilloma virus is the causative agent of


A. Measles
B. Chancre
C. Warts
D. Herpes
113. RNA directed DNA polymerase is exclusively found only amiong
A. Paramyxovirus
B. Adenovirus
C. Togavirus
D. Retrovirus
114. Quinolones are bactericidal agents which act by
A. Inhibition of beta-lactamase
B. Inhibition of revered transcriptase
C. Inhibition of chromosome supercoiling
D. Inhibition of cell wall synthesis
115. A patient presents with an abscess. The pus was examined containing yellow granules. The sample was
tested negative to acid-fast bacilli. The patient most likely has
A. Nocardiosis
B. Actinomycosis
C. Streptococcal infection
D. Staphylococcal infection
116. The species of shigella responsible for the most severe form of shigellosis
A. S. dysenteriae
B. S. sonnei
C. S. boydii
D. S. flexneri
117. Quelling test is used to test for the presence of
A. Streptococcus pyogenes
B. Staphlococcoss aureus
C. Streptococos pneumoniae
D. Mycoplasma pneumonia
118. A goat farmer presents with a fever of unknown origin, which of the following organism is responsible for
this?
A. B. melitensis
B. B. abortus
C. B. suis
D. Bacillus cereus
119. Majority of coagulase-negative staphylococcal infection cases involving prosthetic heart valve endocarditis
are caused by
A. Staph aureus
B. Staph spidermidis
C. Staph saprophyticus
D. Staph hemolyticus
120. Botulism is brought by
A. Invasion of the gut epithelium by the organism
B. Secretion of an enterotoxin
C. Ingestion of neurotoxin
D. Activation of cyclic AMP
121. A 28 year-old menstruating woman appeared in the emergency room with the following signs and
symptoms; fever, hypotension, a Scarletiniform rash on her trunk, palms, and soles, extreme fatigue and
diarrhea. The patient above most likely has
A. Sacalded skin syndrome
B. Toxic shock syndrome
C. Scarletina
D. Chickenpox
122. A campylobacter-like bacteria that is commonly associated with gastric ulcers
A. Campylobacter jejuni
B. Helicobacter pylori
C. Clostridium difficile
D. Helicobacter fenneliae

Microbiology, Public Health and Immunology


Philippine Association of Colleges of Pharmacy

123. this is the most common form of botulism


A. Infant botulism
B. Foodborne botulism
C. Wound botulism
D. Endemic botulism
124. Which of the following may be used as preventive method in the prevention of botulinum spore germination
in food?
A. Maintaining food in basic pH
B. Storing food at cool temp(8-15 C)
C. Preheat food at 80C for 30 mins
D. Administration of polyvalent antitoxin
125. An anaerobic infection with B. fragilis is characterized by
I. Black discoloration of food containing exudates
II. Susceptibility to penicillins
III. Foul-smelling discharge
A. I only
B. III only
C. II and III only
D. I and III only
E. I,II, II and III
126. A person with boil prepared a snack with mayonnaise for their school field trip. Three of his friends whom he
shared his food became violently ill 2 h after eating the food. What is the most likely cause?
I. Staphylococcus aureus enterotoxin
II. Staphylococcus aureus coagulase
III. Staphylococcus aureus leukocidin
A. I only
B. III only
C. II and III only
D. I and III only
E. I,II, and III
127. Treatment of staph. aureus infection with penicillin is often complicated by
I. Inability of penicillin to penetrate the membrane of stap. Aureus
II. Production of beta-lactamase by Staph. aureus
III. Lack of penicillin binding sites in Staph. aureus
A. I only
B. II inly
C. III only
D. I and III only
E. I,II and III
128. In staphylococci, antibiotic resistance genes can exist on plasmids or chromosomes. The genes are carried
by
I. Prophage
II. Free DNA
III. Transposons
A. I only
B. II inly
C. III only
D. I and III only
E. I,II and III
129. Which of the following statements will differentiate Mycoplasmas from Chlamydiae
I. Ability to grow in artificial media
II. Ability to cause urinary tract infections
III. Susceptibility to penicillins
A. I only
B. II inly
C. III only
D. I and III only
E. I,II and III
130. Brill-Zinsser disease is caused by
A. Rickettsia rickettsia
B. R. prowazekii
C. R. typi
D. R. tsutsugamushi
Microbiology, Public Health and Immunology
Philippine Association of Colleges of Pharmacy

131. a poultry farmer has symptoms of chills, fever and headache. Two weeks ago he lost a large number of his
livestock to an undiagnosed disease. What is the most likely condition of the farmer.
A. Psittacosis
B. Q-fever
C. Anthrax
D. Ornithosis
132. An ill patient denied being bitten by insects. Which of the following Rickettsial infection is most likely the
cause of the illness?
A. rickettsial pox
B. Q-fever
C. Scrub typhus
D. Rocky mountain spotted fever
133. A Mycoplasma which is a common cause of nongonococcal urethritis
A. M. hominis
B. M. pneumonia
C. M. fermentans
D. Ureaplasma urealyticum
134. The distinctive characteristic of Mycoplasmas from other bacteria is their lack of
I. Lipopolysaccharide
II. A cell wall
III. ATP synthesis
A. I only
B. II inly
C. III only
D. I and III only
E. I,II and III
135. Which organism will be most affected by a drug that will inhibit ATP synthesis?
A. Mycoplasma pneumonia
B. Ureaplasma urealyticum
C. R. rickettsia
D. N. gonorrhoeae
136. Which of the following will identify chlamydia trachomatis from chlamydia psittaci?
I. C. trachomatis is sensitive to sulphonamides
II. C. trachomatis has a different lipopolysaccharide antigen
III. C. trachomatis can be stained
A. I only
B. II inly
C. III only
D. I and III only
E. I,II and III
137. This is/are clinical manifestations of C. trachomatis
I. Lymphogranuloma venereum
II. Perinatal conjunctivitis
III. Otitis-media in young children
A. I only
B. II inly
C. III only
D. I and III only
E. I,II and III
138. Which of the following is the causative agent of Cat-scratch disease
A. Chlamydia trachomatis
B. Coxiella burnetii
C. Rickettsia ricketssii
D. Bartonella henselae

139. Which statement(s) explain why beef tapeworm is less serious than pork tapeworm infection?
I. Acute intestinal stoppage is less common in beef tapeworm infection
II. Larval invasion does is not occur in beef tapeworm infection
III. The adult beef tapeworm are smaller
A. I only
Microbiology, Public Health and Immunology
Philippine Association of Colleges of Pharmacy

B. II inly
C. I and II only
D. I and III only
E. I,II and III
140. Which parasitic roundworm hatches in the upper small intestine and releases larves that penetrate the
intestinal wall?
A. Hymenolepis nana
B. Diphyllobothrium latum
C. Fasciola hepatica
D. Ascaris lumbricoides
141. This sexually transmitted protozoan is common among male homosexuals.
A. Giardiasis
B. Amoebiasis
C. Leishmaniasis
D. Trypanosomiasis
142. A patient’s stool examination was found to have a small structures resembling rice grains( proglottids) which
of the following organisms is the most likely present in the patient’s stool?
A. Enterobius vermicularis
B. Ascaris lumbricoides
C. Taenia saginata
D. Trichuris trichuria
143. Humans are both the intermediate and definitive hosts of which parasite?
A. Diphyllobothrium latum
B. Taenia solium
C. Taenia saginata
D. Trichuris trichuria
144. Acute infection of trypanosoma cruzi affects the skin foaming ‘ chagoma’. Chronic stage of disease will form
lesions on which organs.
I. Heart and digestive tract
II. Spleen and pancreas
III. Liver and spleen
A. I
B. II
C. I and II only
D. I and III only
E. I, II and III
145. A malarial organism producing ringlike and crescentlike forms within red blood cells
A. Plasmodium vivax
B. Plasmodium falciparum
C. Plasmodium malariae
D. Plasmodium ovale
146. A fungus formely classified as a parasitic Sporozoan affecting patients with immune deficiency.
A. Microsporidium
B. Cryptosporidium
C. Pneumocystis
D. Blastomyces
147. Diphyllobotrium latum infection may lead to the deficiency of which vitamin?
A. Vit A
B. Vit B9
C. Vit B1
D. VitB12
148. Most macroparasites are extracellular organisms. Which of the following is an intracellular parasite?
A. Enterobius
B. Trichinella
C. Strongloides
D. Ascaris

149. Relationship of animals living together in which one species lives in or on the body of another.
A. Commensalism
B. Mutualism
C. Parasitism
D. Symbiosis
Microbiology, Public Health and Immunology
Philippine Association of Colleges of Pharmacy

150. A type of relationship in which one species of organism uses the body of a larger species as its physical
environment and makes use of that environment to acquire nutrients.
A. Commensalism
B. Mutualism
C. Parasitism
D. Symbiosis
151. A relationship that manifests reciprocity of benefits between the organisms involved
A. Commensalism
B. Mutualism
C. Parasitism
D. Symbiosis
152. A type of relationship that involve one-sided beneficial relationship which usually ending up in a harmful
relationship for one organism.
A. Commensalism
B. Mutualism
C. Parasitism
D. Symbiosis
153. Which of thef following statement(s) is /are true regarding viruses
I. Are cellular organisms, which require host to replicate
II. Contain genetic materials required for production of new viruses
III. Requires the hosts metabolic machinery to transcribe and translate genetic information
A. I
B. II only
C. I and II only
D. II and III only
E. I,II and III
154. This is an example of an anaerobic spore-forming bacilli.
A. Clostridium
B. Bacillus
C. Listeria
D. Mycobacteria
155. Which of the following virus family is characterized as a single stranded DNA, non-enveloped virus
A. Herpesviridae
B. Poxviridae
C. Adenoviridae
D. Parvoviridae
156. This is characterized as the most complex DNA virus
A. Herpesviridae
B. Poxviridae
C. Adenoviridae
D. Parvoviridae
157. This is/are examples of naked DNA viruses
I. papova
II Adeno
III. Parvo
A. I only
B. II only
C. I and II only
D. II and III only
E. I,II and III
157. This is/are examples of enveloped DNA viruses
I. Herpes
II.Hepadna
III.Pox
A. I only E. I, II and III
B. .... II only
C. I and II only
D. II and III only
159. Most of the RNA viruses replicate in the cytoplasm. Which of the following RNA viruses replicate in the
nucleus
A. Retrovirus
B. Togavirus
C. Picorvirus
Microbiology, Public Health and Immunology
Philippine Association of Colleges of Pharmacy

D. Paramyxovirus
160. Generally RNA viruses are single stranded viral organism. Which of the following is the only double-
stranded RNA virus
A. Rhinovirus
B. Coronavirus
C. Reovirus
D. Orthomyxovirus
161. One important generalization for RNA viruses is that they are usually enveloped. Among the RNA viruses there
are three which are naked. Which of the following is/are naked RNA viruses?
I. Picornavirus
II. Reovirus
III. Retrovirus
A. I
B. II only
C. I and II only
D. II and III only
E. I,II and III
162. This is a condition where there are small changes in the antigenic structure of an organism which mat later result
to an epidemic
A. Antigenic shift
B. Antigenic drift
C. Tolerance
D. Immunity
163. This refers to the complete change in the antigenic structure of an organism leading to the synthesis of new
antigenic substances that have never been exposed to the human immune system . This may result to a pandemic
A. Antigenic shift
B. Antigenic drift
C. Tolerance
D. Immunity
164.Which of the influenza viruses is isolated in both man and animals
A. Influenza A
B. Influenza B
C. Influenza C
D. Influenza D
165. this condition that results when children with influenza or varicella are given Aspirin
A. Herxheimer reaction
B. Salicylism
C. Reye syndrome
D. Asthma
166. Amantadine used for influenza A infection has been shown to be effective preventing which step in the viral
replication process?
A. Penetration
B. Uncoating
C. Assembly
D. Release
167. this is/are examples of viruses classified under PAramyxoviridae family
I. influenza virus
II. Respiratory syncitial Virus
III.Measles virus
A. I only
B. II only
C. I and II only
D. II and III only
E. I, II and III

168. A viral organism well known to involve infection of the parotid gland and testes in males.
A. Measles
B. Mumps
C. Herpes virus
D. Respiratory syncyitial virus
Microbiology, Public Health and Immunology
Philippine Association of Colleges of Pharmacy

169. Koplik’s spots seen among patients with measles are usually localized in the
A. Upper Extremities
B. Lower Extremities
C. Mouth
D. Scalp
170. This/these statement/s is/are regarding patients with measles during pregnancy
I. may result to birth defects
II. is associated with spontaneous abortions
III. may also result to premature labor
A. I only
B. II only
C. I and II only
D. II and III only
E. I, II and III
171. Which of the following is/are transmitted via fecal oral route?
I. Hepa A
II. Hepa B
III.Hepa C
A. I only
B. II only
C. I and II only
D. II and III only
E. I, II and III
172. Which of the following Hepavirus is/are example of RNA virus
I. Hepa A
II. Hepa B
III.Hepa C
A. I only
B. II only
C. I and II only
D. II and III only
E. I, II and III
173. Chickenpox is a common disease of children. It is caused by
A. Pox viridae
B. Varicella
C. Zoster
D. CMV
174. Virus which is associated with Burkitt’s lymphoma
A. EMV
B. CMV
C. Varicella-Zoster
D. HSV-1
175. The first intestinal microorganism to be observed under a microscope is
A. Entamoeba histolytica
B. Cryptosporidium parvum
C. Ascaris lumbricoides
D. Giardia lamblia
176. Which of the following species can be found in the small intestine
A. Entamoeba histolytica
B. Giardia lmablia
C. Trichuris trichuria
D. Enterobius vermicularis
177. This organism is one of the important cause of severe darrhea in immunocompromised individuals such as
patients with AIDS
A. Entamoeba histolytica
B. Giardia lamblia
C. Isospora belli
D. Strongyloides stercolis
179. The most common and deadliest plasmodia
A. Plasmodium vivax
B. Plasmodium ovale
C. Plasmodium falciparum
D. Plasmodium malariae
Microbiology, Public Health and Immunology
Philippine Association of Colleges of Pharmacy

180. The organism is the causative agent of erythema infectionism in children


A. Adenovirus
B. Parvovirus
C. Polymavirus
D. Cytomegalovirus
181. Which of the following belong/s to the group of the ARBOvirus?
I. Bunyaviridae
II. Falviviridae
Coronaviridae
A. I only
B. II only
C. I and II only
D. II and III only
E. I, II and III
182. The presence of this in the serum patients indicates immunity to Hepatitis B virus.
A. HBsAg
B. Anti HBsAg
C. Anti HBcAg
D. Anti HBeAg
183. The presence of theis antigen indicates high infectivity and active disease of Hepa B virus
A. HBsAg
B. HBeAg
C. HBcAg
D. Anti HBeAg
184.The presence of this in the serum of the patient with hepatitis will ascertain that the patient have an old Hepatitis
B infection
A. Anti HBsAg
B. IgM-Anti HBcAg
C. IgG-Anti HBcAg
D. Anti HBeAg
185. This is a parentallu transmitted(Blood-borne) hepatitis which can only replicate and cause infection with help of
Hepatitis B virus
A. HEV
B. HAV
C. HCV
D. HDV
186. This/ These statement/s is/are true for HSV-1
I. Causes gingivostomatitis
II. Causes herpetic keratitis
III. An STD
A. I only
B. II only
C. I and II only
D. II and III only
E. I, II and III
187. This is/are example of organisms that can cross the placenta
I. Rubeola
II. Cytomegalovirus
III. Leprosy
A. I only
B. II only
C. I and II only
D. II and III only
E. I, II and

188. This is the ARBOvirus family from which the dengue fever and yellow fever virus are classified
A. Bunyaviridae
B. Togaviridae
C. Flaviviridae
D. Caliciviridae
Microbiology, Public Health and Immunology
Philippine Association of Colleges of Pharmacy

189. Which of the following viral organicm is characteristically associated with Peyer’s patchesof the intestines
A. Poliovirus
B. Rubella
C. Coronavirus
D. Adenovirus
190. This/These statement/s is/are true concerning the polio vaccine
I. Salk vaccine is made of killed-polio viruses
II. Sabin vaccine is made of attenuated polio virus
III. The vaccine can pick up virulence and cause paralysis
A. I only
B. II only
C. I and II only
D. II and III only
E. I, II and III
191. This/ These is/are the common viral organism/s
I. Rhinovirus
II. Coronavirus
III. Flavivirus
A. I only
B. II only
C. I and II only
D. II and III only
E. I, II and III
192. Which viral organism has a characteristic bullet-shaped, enveloped, helical nucleocapsid?
A. Poliovirus
B. Rabies virus
C. Coronavirus
D. Adenovirus
193. This is/are statement(s) pertaining to the rabies virus
I. Brain cells of infected animals and humans contain virions in the cytoplasm called Negri bodies
II. Bitten individual should receive human rabies immune globulin(passive immunization)
III. Killed rabies virus vaccine (active immunization) is given only as prophylaxis for rabies.
A. I only
B. II only
C. I and II only
D. II and III only
E. I, II and III
194. This is also known as the “break bone fever”
A. Dengue fever
B. Yellow fever
C. Hay fever
D. Relapsing fever
195. Protozoa are single-celled eukaryotic organisms. They may be classified according to their organ for
locomotion. Which of the following protozoan has no definite organ for motility
A. Sarcodina
B. Mastigophora
C. Ciliaphora
D. Sporozoa
196. Member(s) of the phylum Apicomplexa with motile zygote
I. Cryptosporidium
II. Plasmodium
III. Toxoplasma
A. I only
B. III only
C. I and II only
D. II and III only
E. I, II and III

197. This intestinal protozoan/s cause/s severe diarrhea in individuals with defective immune system such as
patients with AIDS
I. Giardia
II. Isospora
III. Crystosporidium
Microbiology, Public Health and Immunology
Philippine Association of Colleges of Pharmacy

A. I only
B. III only
C. I and II only
D. II and III only
E. I, II and III
198. This is the ingested infective form of protozoan parasites in humans
A. Cysts
B. Merozoites
C. Trophozoites
D. Cytostome
199. This is/are examples of free-living pathogenic amoebae in man
I. Entamoeba
II. Naegleria
III. Acanthamoeba
A. I only
B. III only
C. I and II only
D. II and III only
E. I, II and III
200. A protozoan which has high incidence of infection among campers and backpackers after drinking from “clear”
mountain streams.
A. Entamoeba
B. Trychomonas
C. Giardia
D. Toxoplasma
201. This is an experimental drug given for AIDS patients suffering from Cryptosporidiosis
A. Spiramycin
B. Cotrimoxazole
C. Albendazole
D. Amphotericin B
202. This is the drug of choice in AIDS patients suffering from Isospora belli infection\
A. Spiramycin
B. Cotrimoxazole
C. Albendazole
D. Amphotericin B
203. This is the causative agent of the so called”Leningrad’s curse”
A. Entamoeba
B. Trychomonas
C. Giardia
D. Toxoplasma
204. A protozoan organism usually opportunistic among patients with AIDS. It is naturally harboured by cats
A. Entamoeba
B. Trychomonas
C. Giardia
D. Toxoplasma
205. This is the most common opportunistic infection among AIDS patients
A. Cryptosporidiosis
B. Isosporosis
C. Pneumonia
D. Toxoplasmosis
206. This drug is given both as a prophylactic and symptomatic treatment of PCP in AIDS patients.
A. Spiramycin
B. Cotrimoxazole
C. Albendazole
D. Amphotericin B
207. this is the malarial species that causes the called ”Quartan malaria”
A. P.ovale C. P.malariae
B. P.vivax D. P.falciparum
208. This is/are true statements regarding malaria
I. Schizogeny or the asexual cycle of malaria occurs in mosquito
II. Sporogeny or the sexual cycle of malaria occurs in man
III. Mosquitoes are the definitive hosts for malaria
A. I only
Microbiology, Public Health and Immunology
Philippine Association of Colleges of Pharmacy

B. III only
C. I and II only
D. II and III only
E. I, II and III
209. This is the malarial species that has a characteristic crescent or banana shaped gametocytes
A. P.ovale
B. P.vivax
C. P.malariae
D. P.falciparum
210. The release of this from of the malarial parasite triggers the fever, chills and sweats of malaria
A. Sporozoites
B. Merozoites
C. Trophozoites
D. Gametocytes
211. This is/are the species of malaria which are capable of forming dormant in the liver called the hypnozoites
I. P. vivax
II. P.falciparum
III. P.ovale
A. I only
B. III only
C. I and II only
D. II and III only
E. I, II and III
212. This is a form of the malarial parasite that is taken up ny the mosquito as it bites on human
A. Sporozoites
B. Merozoites
C. Trophozoites
D. Gametocytes
213. Which of the following drugs is given for the exo-erythrocytic form of malaria?
A. Primaquine
B. Chloroquine
C. Mefloquine
D. Doxycycline
214. This is/are agents given for Chloroquine-resistant P. falxiparum
I. Quinine
II. Fansidar
III. Mefloquine
A. I only
B. III only
C. I and II only
D. II and III only
E. I, II and III
215. This is the drug coined used as the “radical cure” of malaria
A. Primaquine
B. Chloroquine
C. Mefloquine
D. Doxycycline
216. This is the only form of the blood-borne flagellates which is non-flagellated
A. Promastigote
B. Epimastigote
C. Amastigote
D. Trypomastigote
217. This is the organism which causes kala-azar or Viscerla Leishmaniasis
A. L.braziliensis
B. L.donovani
C. L.tropica
D. L.cruzi

218. This is the causative agent of Espundia or Mucocutaneous leishmaniasis


A. L.braziliensis
B. L.donovani
C. L.tropica
D. L.cruzi
Microbiology, Public Health and Immunology
Philippine Association of Colleges of Pharmacy

219. This is thedrug of choice for all forms of leishmaniasis


A. Primaquine
B. Stibogluconate
C. Pentamidine
D. Suramin
220. This is the causative agent of the West African sleeping disease
A. Trypanosome brucei rhodesiense
B. T.brucei gambiense
C. T.cruzi
D. L.donovani
221. this is the drug given for the treatment of African sleeping disease
A. Nifrutimox
B. Stibogluconate
C. Sulfadiazine
D. Suramin
222. There is currently no approved drug for the treatment of American sleeping disease, although there are two
drugs currently used for acute cases only. Which of the following drugs is used for acute cases of American sleeping
disease?
A. Nifrutimox
B. Stibogluconate
C. Sulfadiazine
D. Suramin
223. This is/ are examples of nematode that can cause infection by penetrating intact skin
I. Necator americanus
II. Trichuris trichura
III. Enterobius vermacularis
A. I only
B. III only
C. I and II only
D. II and III only
E. I, II and III
224. This is/ are examples of nematodes which is/are acquired thru the ingestion of eggs
I. Ascaris lumbricoides
II. Trichuris trichura
III. Strongyloides stercoralis
A. I only
B. III only
C. I and II only
D. II and III only
E. I, II and III
225. This nematode is acquired through the ingestion of encysted larvae present in pork meat.
A. Ascaris lumbricoides
B. Trichuris trichura
C. Strongyloides stercoralis
D. Trichinella spiralis
226. This is the drug of choice for patients having Ascariasis
A. Pyrantel pamoate
B. Invermectin
C. Mebendazole
D. Praziquantel
227. This is the drug of choice for patients having filriasis
A. Thiabendazole
B. Ivemectin
C. Mebendazole
D. Praziquantel

228. Drug of choice for patients suffering from strongyloides sterecularis infection
A. Thiabendazole
B. Ivemectin
C. Mebendazole
D. Praziquantel
Microbiology, Public Health and Immunology
Philippine Association of Colleges of Pharmacy

229.This is the drug of choice for flukes


A. Thiabendazole
B. Ivemectin
C. Mebendazole
D. Praziquantel
230. The infection from this nematode will cause a rise in the levels of serum muscle enzymes such as creatinine
phosphokinase
A. Ascaris lumbricoides
B. Trichinella spiralis
C. Strongyloides stercoralis
D. Trichuris trichura
231. A helminth infection usually is accompanied by increase in the serum levels of eosinophils. This may be result
of the tissue invasion of the helminth. Which of the following organisms listed has no tissue invasion, hence, will not
manifest with elevated level of serum eosinophils.
A. Ascaris lumbricoides
B. Trichinella spiralis
C. Strongyloides stercoralis
D. Trichuris trichura
232. This is a diagnostic test for the presence of Enterobius vermicularis
A. Kato thick method
B. Kato-katz method
C. Scotch tape swab method
D. Brine flotation technique
233. This is/are example(s) of nematode which is/are transmitted usually by an arthropod vector.
I. Onchocerca volvulus
II. Wuchereria bancrofti
III. Brugia malayi
A. I only
B. III only
C. I and II only
D. II and III only
E. I, II and III
234. This organism is usually acquired by fishermen eating raw small brackfish water fish called “bagsit”
( Hypseleotris bipartite )
A. Ascaris lumbricoides
B. Trichinella spiralis
C. Capillaria philippinensis
D. Trichuris trichura
235. This drug is given as the drug of choice for the bancroftian filariasis
A. Diethylcarbamazine
B. Ivermectin
C. Mebendazole
D. Praziquantel
236. This is an example of trematode which inhibit/s the portal blood stream of vertebrates
A. Fasciola hepatica
B. Schistosoma japonicum
C. Fasciolopsis buski
D. Clonorchis sinensis
237. This is/are example/s of flukes which inhibit/s the liver if vertebrates
I. Fasciola hepatica
II. Clonorchis sinensis
III. Paragonimus westermani
A. I only
B. III only
C. I and II only
D. II and III only
E. I, II and III

238. This is/are example/s of tapeworms which require/s a vertebrate as intermediate host
I. Taenia solium
II. Hymenolepsis nana
III. Dipylidium caninum
A. I only
Microbiology, Public Health and Immunology
Philippine Association of Colleges of Pharmacy

B. III only
C. I and II only
D. II and III only
E. I, II and III
239. Which of the following tepeworms will require an invertebrate as intermediate host?
I. Taenia solium
II. Hymenolepsis nana
III. Dipylidium caninum
A. I only
B. III only
C. I and II only
D. II and III only
E. I, II and III
240. This is the organism which cause the Chinese liver fluke infection
A. Fasciola hepatica
B. Clonorchis sinensis
C. Fasciolopsis buski
D. Schistosoma japonicum
241. This is also known as the Giant intestinal fluke
A. Fasciola hepatica
B. Clonorchis sinensis
C. Fasciolopsis buski
D. Schistosoma japonicum
242. this is/are true statement/s regarding Schistosoma
I. They are the number one worldwide water-borne protozoal infection
II. They multiply in humans
III. Eggs must reach freshwater to hatch
A. I only
B. III only
C. I and II only
D. II and III only
E. I, II and III
243. There are different species of Schistosoma and common in them is they lay eggs in the feces. Which of the
following Schistosoma deposits eggs in the urine?
A. Schistosoma heamatobium
B. Schistosoma Israeli
C. Schistosoma mansoni
D. Schistosoma japonicum
244. Aside from Praziquantel, this drug is also given for tapeworms
A. Diethylcarbamazine
B. Ivermectin
C. Mebendazole
D. Niclosamide
245. this is the smallest tapeworm that infects humans
A. Taenia solium
B. Taenia saginata
C. Hymenolepsis nana
D. Echinococcus granulosus
246. This is the organism causing the so called River blindness
A. Ascaris lumbricoides
B. Trichinella spiralis
C. Capillaria philippinensis
D. Onchocerca volvulus
247. This is the organism causing “Hydratid disease”
A. Taenia solium
B. Taenia saginata
C. Hymenolepsis nana
D. Echinococcus granulosus

248. This is/are example/s of antihelminthic drugs which produce/s their effects by inducing paralysis of the worm
I. Praziquantel
II. Mebendazole
III. Diethylcarbamazine
Microbiology, Public Health and Immunology
Philippine Association of Colleges of Pharmacy

A. I only
B. III only
C. I and II only
D. II and III only
E. I, II and III
249. This is/ are example/s of filarial nematodes
I. Wuchereria bancrofti
II. Brugia malayi
III. Loa loa
A. I only
B. III only
C. I and II only
D. II and III only
E. I, II and III
250. Endotoxins are usually found in garam-neagative organism. Which of the is the only gram-positive organism
which produces an endotoxin
A. Listeria monocytogenes
B. Bacillus anthracis
C. Clostridium sp
D. Nocardia
251. This is the average time required for the population , or the biomass of an organism to double
A. Lag time
B. Growth rate constant
C. Generation time
D. Priming time
252. This is the portion in the bacterial growth curve representing the period at which the newly inoculated cells are
adapting to their new environment. Enzymes and intermediates are formed and accumulated until they are present
inn concentrations that permit growth to resume.
A. Lag time
B. Exponential phase
C. Maximum stationary phase
D. Death phase
253. This phase in the bacterial growth curve represents the time where exhaustion of nutrients or the accumulation
of toxic products causes growth to cease completely. There is slow loss of cells through death, which is just
balanced by the formation of new cells through growth and division. This is a period where the total count slowly
increases although the variable count stays constant.
A. Lag time
B. Exponential phase
C. Maximum stationary phase
D. Death phase
254. This is/are example/s of drugs that cause/s the destruction of bacterial DNA
I. Ionizing radiation
II. Alkylating agents
III. UV light
A. I only
B. III only
C. I and II only
D. II and III only
E. I, II and III
255. Which of the following is the mechanism of action of Amphotecin B as an antifungal agent/
A. Inhibition of cell wall synthesis
B. Alteration of cell membrane function
C. Inhibition of protein synthesis
D. Inhibition of nucleic acid synthesis

256. This is/are possible ways in which bacteria may gain resistance to antibacterial agents
I. Alteration of the target site
II. Synthesis of enzymes that modify or destroy the antibacterial agent
III. Presence of capsule inn a bacteria
A. I only
Microbiology, Public Health and Immunology
Philippine Association of Colleges of Pharmacy

B. III only
C. I and II only
D. II and III only
E. I, II and III
257. This is/are example/s of drugs that inhibit/s the cell wall synthesis
I. Penicillins
II. Cyclosporine
III. Teicoplanin
A. I only
B. II only
C. I and II only
D. I and III only
E. I, II and III
258. This/ These statement/s is/are true about the glycopeptide antibiotics
I. Given only parentally as it is not absorbed from the GIT
II. Given only for gram-positive organism
III. Potentially ototoxic and nephrotoxic
A. I only
B. II only
C. I and II only
D. I and III only
E. I, II and III
259. This is/are aminoglycoside antibiotics derived from Streptomyces species
I. Streptomycin
II. Gentamicin
III. Azithromycin
A. I only
B. II only
C. I and II only
D. I and III only
E. I, II and III
260. Tetracyclines are indicated for the treatment of bacterial infection from which organism/s
I. Staphylococcus
II. Mycoplasma
III. Chlamydia
A. I only
B. III only
C. I and II only
D. II and III only
E. I, II and III
261. This antibacterial agent claimed its glory by being the first broad spectrum antibiotic introduced. It contains a
nitrobenzene nucleus, which is responsible for some of its toxic side effects. It blocks the action of peptidyl
transferase, thereby preventing peptide bond synthesis. Currently its main indication is for infections of Salmonella
typi
A. Erythromycin
B. Doxycycline
C. Chloramphenicol
D. Vancomycin
262. This is the most common drug used as alternative for patients allergic to penicillins
A. Erythromycin
B. Doxycycline
C. Chloramphenicol
D. Vancomycin
263. This is a steroid-like compound that inhibit protein synthesis by forming a atable complex with elongation factor
EF-G , guanosine diphosphate and the ribosome. It is given for gram-positive cocci but not used alone due to the
development of rapid resistance to the drug.
A. Erythromycin C. sulfonamides
B. Fusidic acid D. lincomycin
264. sulphonamides are drugs acting as nucleic acid synthesis inhibitors by competing this substrate
A. Aminoacyl C. PABA
B. ATP D. Peptidyl tRNA
265. This is/are example/s of drugs which are structural analogues of the aminohyroxypyrimidine amoiety of folic
acid prevent/s its synthesis to THFS.
Microbiology, Public Health and Immunology
Philippine Association of Colleges of Pharmacy

I. Trimethoprim
II. Pyrimethamine
III. Methotrexate
A. I only
B. III only
C. I and II only
D. II and III only
E. I, II and III
266. These antibacterial agents prevent the supercoiling of the bacterial chromosomes
A. Macrolides
B. Rifampicin
C. Nalidixic acid
D. Clindamycin
267. these drugs are contraindicated in children due to the possible toxic effects on cartilage development
A. Ciprofloxacin
B. Rifampicin
C. Metronidazole
D. Clindamycin
268. This drug acts by inhibiting the synthesis of mRNA
A. Ciprofloxacin
B. Rifampicin
C. Metronidazole
D. Clindamycin
269. This drug is currently given as the drug of choice for the prophylaxis of close contacts of meningococcal and
Haemophilus meningitis
A. Ciprofloxacin
B. Rifampicin
C. Metronidazole
D. Clindamycin
270. This drug is important in the management of parasites and anaerobic bacteria
A. Rifabutin
B. Rifampicin
C. Metronidazole
D. Nitrofrantoin
271. This is/are true statements about Colistin
I. It is a polymyxin antibiotic acting on the cell membrane
II. It is systemically given for systemic infections
III. It is used topically for wound irrigation and bladder washout
A. I only
B. III only
C. I and II only
D. I and III only
E. I, II and III
272. Methenamine is used as urinary tract antiseptic/s. Methenamine is hydrolysed into which compound to provide
its antibacterial effect in the urinary tract?
A. Ethylene
B. Formaldehyde
C. Ammonia
D. Acetic acid
273. This is given together with isoniazid for patients with TB to prevent its neurologic side effects
A. Rifampicin
B. Ethambutol
C. Pyridoxine
D. Pyrazinamide
274. This is anti-TB drug that inhibits and does not kill mycobacterium tuberculosis. It usually presents an important
toxic side effects such as optic neuritis
A. Rifampicin C. Pyridoxine
B. Ethambutol D. Pyrazinamide
275. This expensive drug is given for dapsone-resistant M.leprae
A. Thalidomide
B. Para-aminosalicylic acid
C. Clofazime
D. Rifampicin
Microbiology, Public Health and Immunology
Philippine Association of Colleges of Pharmacy

276. This is the most important agent used for severe systemic mycoses, but is toxic
A. Ketoconazole
B. Miconazole
C. Flucytosine
D. Amphotericin
277. This is an antifungal that acts by inhibiting lanosterol C14-demethylase, an important enzyme in sterol
synthesis
A. Griseofulvin
B. Miconazole
Flucytosine
C. Amphotericin
278. This is an important antifungal agent which inhibits DNA synthesis and is active only on yeasts (candida and
Cryptococcus )
A. Griseofulvin
B. Miconazole
C. Flucytosine
D. Amphotericin
279. An antifungal that inhibits nucleic acid synthesis and has anti-mitotic activity by inhibiting microtubule assembly
A. Griseofulvin
B. Miconazole
C. Flucytosine
D. Amphotericin
280. This is an example of a topical agent, which is derived from the combination of benzoic and salicylic acids.
A. Haloprogin
B. Tolnaftate
C. Naftifine
D. Whitfield’s ointment
281. This antiviral agent specifically acts by inhibiting the penetration and uncoating of viral organism
A. Acyclovir
B. Amantadine
C. Ribavirin
D. Interferons
282. This is an antiviral agent which is effective in preventing vial protein synthesis
A. Acyclovir
B. Amantadine
C. Ribavirin
D. Interferons
283. This is/are example of antiviral agent/s that act in the viral DNA or RNA synthesis
I. Idoxuridine
II. Acyclovir
III. Zidovudine
A. I only
B. III only
C. I and II only
D. I and III only
E. I, II and III
284. Vaccines are prepared from which of the following
I. Immunoglobulins
II. Killed organism
III. ‘Subcellular fractions
A. I only
B. III only
C. I and II only
D. II and III only
E. I, II and III

285. This was the first vaccine in human used to be made by recombinant DNA technology
A. Polio vaccine
B. Rabies vaccine
C. Hepa B
D. Measles vaccine
Microbiology, Public Health and Immunology
Philippine Association of Colleges of Pharmacy

286. This is the only viral disease where post-exposure vaccination is successful due to its long incubation period
A. Polio
B. Rabies
C. Hepa B
D. Measles
287. This is an agent used for the control of bacterial growth, and is classified as an alkylating agent
A. Formaldehyde
B. Alcohols
C. Detergents
D. Heavy metals ions
288. This agent works by binding to sulfhydryl groups of bacteria
A. Formaldehyde
B. Alcohols
C. Detergents
D. Heavy metals ions2
289. These organisms do not require organic nutrients for growth.
A. Heterotrophs
B. Chemolithotrophs
C. Autotrophs
D. Psychotrophs
290. These are organisms that use an inorganic substrate as hydrogen or thiosulfate as reductant and carbon
monoxide as carbon source.
A. Heterotrophs
B. Chemolithotrophs
C. Autotrophs
D. Psychotrophs
291. These are organisms characterized to have flagella all over the cell body
A. Lopotrichous
B. Atrichous
C. Peritrichous
D. Amphitrichous
292. These are organisms that possess no flagella
A. Lopotrichous
B. Atrichous
C. Peritrichous
D. Amphitrichous
293. These are organisms that possess a tuft of flagella at one end
A. Lopotrichous
B. Atrichous
C. Peritrichous
D. Amphitrichous
294. There are generally two types of stain ,a n Acidic and Basic stain. Which is/are true statement/s about Basic
stains
I. They are composed of colored cation with a colorless anion
II. They combine with negatively charged phosphate present in bacterial cells
III. They are used in negative staining
A. I only
B. III only
C. I and II only
D. I and III only
E. I, II and III
295. This is a specific stain used for staining nuclei
A. Feulgen stain
B. Carbofuchsin
C. Nigrosin

296. This is also known as the Welch method


A. Gram stain
B. Acid-fast stain
C. Spore stain
D. Capsule stain
Microbiology, Public Health and Immunology
Philippine Association of Colleges of Pharmacy

297. A stain specifically used to stain spores


A. Malachite green
B. Carbolfuchsin
C. Nigrosin
D. Methylene blue
298. First proposed the theory of Biogenesis, which states that life must arise from pre-existing life
A. Louis Pasteur
B. John Tyndall
C. Rudolf Virchow
D. Lazaro Spallanzani
299. Developed the method of producing immunity by using antitoxin against diphtheria
A. Joseph Lister
B. Emil von Behring
C. Robert Koch
D. Paul Ehrlich
300. A German physician given credit for developing pure-culture techniques
A. Joseph Lister
B. Emil von Behring
C. Robert Koch
D. Paul Ehrlich

Microbiology, Public Health and Immunology


MICROBIOLOGY & PUBLIC HEALTH

A 1. The sudden unexpected occurrence of a disease in a given population:


A) Outbreak B) Zoonotic C) Endemic D) Sporadic

D 2. A causative agent of peptic ulcer:


A) Escherichia coli B) Bordetella pertussi C) Shigella dysenteriae D) Helicobacter pylori

A 3. The “four o’clock habit” promoted by the Department of Health is designed to prevent the spread of:
A) Dengue fever B) Tuberculosis C) Malnutrition D) Malaria

A 4. The first drug available for HIV:


A) ZDU B) RMP C) TMP D) CMC

B 5. An immediate hypersensitivity reaction following exposure of a sensitized individual to the appropriate


antigen:
A) Hay fever B) Anaphylaxis C) Asthma D) Desensitization

D 6. A thin proteinaceous appendage necessary for bacterial conjugation:


A) Cilium B) Trichome C) Flagellum D) Pilus

B 7. The use of nitrite as preservative for food is discouraged because:


A) It can discolor the meat
B) It can react with amines to form carcinogenic nitrosamines
C) It decomposed to nitric acid which can react with heme pigments
D) It is not readily available

B 8. The disinfectant of choice for municipal water supplies:


A) Lysol B) Chlorine C) Ozone D) Reverse osmosis

B 9. An object that is able to harbor and transmit microorganisms:


A) Mite B) Fomite C) Arthropod D) Vector

C 10. Administration of a toxoid confers:


A) Naturally acquired active immunity C) Artificially acquired active immunity
B) Naturally acquired passive immunity D) Artificially acquired passive immunity

A 11. An index that measures the number of individuals who have become ill because of a specific disease
within a susceptible population during a specific period:
A) Morbidity rate B) Prevalence rate C) Mortality rate D) All of the above

D 12. The effectiveness of a disinfectant is influenced by:


A) Population size C) Concentration of the disinfectant
B) Duration of exposure D) All of the above

A 13. Lesions in the oral cavity caused by measles virus are known as:
A) Koplik spot B) Peyer’s pathches C) Rose spots D) Rashes

C 14. Anaerobic bacteria that derive energy by converting formates, acetates and other compounds to
methane:
A) Microaerophilic B) Metanochromic C) Methanogenic D) Cyanobactreria

B 15. Artificially acquired passive immunity is developed after vaccination with:


A) Attenuated microorganism C) Toxoids
B) Immunoglobulin preparations D) Cyanobacteria

A 16. Nosocomial infections are:


A) Infections developed while the patient is in the hospital
B) Infections of the nasal area
C) Infections where pathogens enters the body through the nose
D) Infections among animals

C 17. The capacity of an organism to produce a toxin is known as:


A) Pathogenicity B) Virulence C) Toxigenicity D) Toxicity

B 18. The Boracay water was declared unsafe due to:


A) Industrial wastes B) Fecal coliforms C) Red tide D) Oil spills

D 19. The male ascaris is distinguished from the female because its tail is:
A) Straight B) Slim C) Blunt D) Curved

A 20. It refers to water suitable for drinking:


A) Potable B) Edible C) Bacteria-free D) Odor-free

B 21. Microbial decomposition of proteins with the production of H2S and amines is known as:
www.brex.us 279
MICROBIOLOGY & PUBLIC HEALTH

A) Fermentation B) Putrefaction C) Dentrification D) Transpeptidation

D 22. Rod-shaped bacteria curved to form commas are known as:


A) Bacilli B) Cocci C) Spiral D) Vibrios

A 23. This statement is true about viral infection:


A) Viral infections are self-limiting
B) Viral infections confer lifetime immunity
C) Viral infections are treated by antibiotics
D) All statements are correct

D 24. The heat-stable lipopolysaccharide in the outer membrane of gram-negative cell wall that is toxic to the
host is known as:
A) Tetanospasmin B) Aflatoxin C) Enterotoxin D) Endotoxin

A 25. The ability of a microorganism to enter a host, grow, reproduce and spread throughout its body is known
as:
A) Invasiveness B) Pathogenicity C) Toxigenicity D) Virulence

D 26. The intimate living together of members of two different species is known as:
A) Mutualism B) Commensalism C) Symbiosis D) All of the above

B 27. Plastics that can be decomposed by microorganism are said to be:


A) Recyclable B) Biodegradable C) Earth-friendly D) Reversible

B 28. Dengue virus vector:


A) Plasmodium falciparum B) Aedes aegypti C) Anopheles mosquito D) None of the above

D 29. It causes ringworm infection with whitish patches on human skin:


A) Tinea capitis B) Tines corporis
C) Trichophyton rubrum D) Malasseria furfur

D 30. The causative agent of pneumonia acquired by inhalation from air-conditioners:


A) Chlamydia pneumoniae B) Salmonella typhi
C) Pneumocystis carinii D) Legionella pneumophila

D 31. The following are communicable diseases, except:


A) Measles B) Pneumonia C) Hepatitis D) Tetanus

A 32. The following are zoonotic diseases, except:


A) Mumps B) Leptospirosis C) Anthrax D) Brucellosis

A 33. Hansen’s disease is caused by:


A) Mycobacterium leprae B) Mycobacterium tuberculosis
C) Myoplasma leprae D) None of the above

B 34. The following are true about moist heat sterilization, except:
A) Makes use of an autoclave C) Makes use of steam under pressure
B) Causes oxidation of cell components D) Can kill both vegetative cells and spores

D 35. A cellular structure equated for drug resistance is the:


A) Endospore B) Exospore C) Transposon D) Capsule

D 36. Bacterial genes responsible for drug resistance are known as:
A) F plasmids B) R plasmids C) Transposons D) Both B and C

B 37. Group of microorganisms that can be transmitted to humans by animal vectors like insects:
A) Protozoa B) Rickettias C) Chlamydias D) Myoplasmas

C 38. The first chemotherapeutic agent scientifically discovered and evaluated:


A) Sulfonamide B) Penicillin C) Salvarsan D) None of the above

C 39. Thioglycollate agar is an example of:


A) Enriched media B) Selective media C) Anaerobic media D) Differential media

B 40. A chemical agent that kills the vegetative forms of pathogenic microorganisms but not necessarily the
spores is known as:
A) Antiseptic B) Disinfectant C) Sanitizer D) Germicide

A 41. An example of ionizing radiation:


A) X-ray B) UV C) Sunlight D) All of the above

D 42. The primary site of electron transport system in eukaryotes:


www.brex.us 280
MICROBIOLOGY & PUBLIC HEALTH

A) Ribosomes B) Nucleus C) Cytoplasm D) Mitochondria

A 43. The organism with atypical cell walls:


A) Mycoplasmas B) Rickettias C) Chlamydias D) Viruses

A 44. Koch’s postulates include the following, except:


A) The suspected organism should be present in health individuals
B) The organism must be isolated and grown as pure culture in the laboratory
C) The organism must initiate the symptoms of the disease in healthy animals
D) The organism must be re-isolated from the animal and be cultured again in the laboratory

B 45. The following are contributions of Louis Pasteur in microbiology, except:


A) Terms “aerobic” and “anaerobic” C) Pasteur flask
B) Fractional sterilization D) Fermentation

C 46. A phenomenon wherein an organism exhibits plasticity:


A) Dimorphism B) Polymorphism C) Pleomorphism D) Fleximorphism

A 47. The following can be sterilized in an autoclave, except:


A) Olive oil B) Culture media C) Water D) Glassware

C 48. A biological sterilization indicator used to test autoclave efficiency:


A) Virus B) Bacillus thuringiensis
C) Bacillus stearothermophilus D) Clostridium botulinum

C 49. Phase in the bacterial growth curve wherein the culture is in the period of balanced growth:
A) Log phase B) Lag phase C) Stationary phase D) Death phase

C 50. A process by which bacterial endospore returns to its vegetative state:


A) Polymorphism B) Mutagenicity C) Germination D) Sporulation

B 51. Ribosomes of Candida albicans are referred to as:


A) 70s B) 80s C) 90s D) 100s

D 52. Organisms that can grow at body temperature:


A) Thermophiles B) Psychrophiles C) Acidophiles D) Mesophiles

B 53. Specific process of reproduction among prokaryotes:


A) Sporulation B) Transverse binary fission C) Germination D) None of the above

A 54. A process of gene transfer where the carrier of DNA is a virus:


A) Transduction B) Conjugation C) Transformation D) Both A and C

A 55. A structurally mature infections viral particle:


A) Virion B) Capsid C) Capsule D) Spore

A 56. An algae group responsible for red tide poisoning:


A) Dinoflagellate B) Diatom C) Euglenoid D) None of the above

A 57. A theory that states that life originates from non-life:


A) Theory of Spontaneous Generation C) Germ Theory
B) Koch’s Postulates D) None of the above

A 58. Lymphocytes that have a high affinity for HIV:


A) T-helper lymphocytes B) B-lymphocytes C) Phagocytes D) T-cytotoxic lymphocytes

C 59. A priority program of DOH, which aims at promoting availability of quality services in health centers and
hospitals:
A) Health Sector Reform Agenda D) Health Passport Initiative
B) National Health Objectives E) None of the above
C) Sentrong Sigla

B 60. Another priority program of DOH, which emphasizes partnership and shared responsibility for health
among various sectors:
A) Health Sector Reform Agenda D) Health Passport Initiative
B) National Health Objectives E) None of the above
C) Sentrong Sigla

A 61. Its function is mainly to serve as an advisory body to the local executive or local legislative on health
related matters:
A) DOH B) WHO C) QUERT D) Sentrong Sigla E) None of the above

www.brex.us 281
MICROBIOLOGY & PUBLIC HEALTH

D 62. Backyard gardening, community project and putting of herbal plants are among the major activities under
this program:
A) Araw ng Sangkap Pinoy D) WHO
B) Nutrition Program E) None of the above
C) National Drug Policy Program

D 63. Its main objective is attainment by all people of the highest possible level of health:
A) Philhealth B) DOH Hospitals C) UNICEF D) WHO E) None of the above

D 64. National Center for Disease Prevention and Control belongs to what function cluster:
A) Internal Management D) Health Regulation Development
B) Health Regulation E) None of the above
C) External Affairs

B 65. An employee liability law that provides financial supports for workers unemployed because of work
related injuries:
A) OSH Act D) Coal Mine Health and Safety Act
B) Workers Compensation Law E) None of the above
C) Civilian Rehabilitation

E 66. A lung disease caused by cotton-mill dust:


A) Mesothelioma C) Physical hazards E) None of the above
B) Chemical hazards ` D) Ergonomic hazards

C 67. A type of industrial hazard, which includes ambient heat, burn, noise and vibration:
A) Biological C) Physical E) None of the above
B) Chemical D) Ergonomic

B 68. Air contaminants causing death by asphyxiation in high concentration:


A) Sulfur dioxide B) CO C) CO2 D) Lead E) None of the above

E 69 The following are functions of Amiotic fluid, except:


A) Allows the movement of fetus D) Allows the fetus to float
B) Protects against mechanical injury E) None of the above
C) Provides stable temperature

A 70. Food processing technique that protects food from oxidative deterioration and growth of aerobic
microorganism:
A) Canning B) Pasteurization C) Irradiation D) Drying E) None of the above

C 71. A vitamin that serves as intracellular antioxidants:


A) Vit. A B) Vit. D C) Vit. E D) Vit. K E) None of the above

C 72. The irreversible stage of alcohol damage characterized by liver enlargement:


A) Cirrhosis B) Alcoholic hepatitis C) Fatty liver D) All E) None

E 73. The following are attributed to fetal alcohol syndrome, except:


A) Smaller size B) Deformities of limb C) Heart defects D) Poor coordination E) None

B 74. Stage of alcoholism wherein the person experiences blackouts:


A) Initial stage B) Middle stage C) Third stage D) final stage E) None

E 75. Compounds that combine with atmospheric moisture to produce highly acidic rain, snow, hair or fog:
A) Carbon dioxide B) Sulfur dioxide C) Nitrogen oxide D) A and B E) B and C

D 76. It is an aging process in the life cycle of lake, pond or slow moving river and stream brought about by the
accumulation of nutrients needed to sustain aquatic plants and animals accompanied by an increase in
the number of organisms:
A) Acidification D) Eutrophication
B) Alkalinification E) None
C) Nitrification
B 77. A drug for treatment of alcoholism, which acts to diminish the pleasurable effects of alcohol:
A) Naltrexone B) Antabuse C) Tranquilizers D) All E) None

A 78. This is a period between conceptions through complete delivery of the product of conception:
A) Pregnancy B) Fertilization C) Implantation D) Cleavage E) None

A 79. A trace mineral necessary for heme synthesis, electron transport and wound healing:
A) Copper B) Manganese C) Chromium D) Zinc E) None

A 80. It describes the amounts of energy, protein, minerals and vitamins needed by normal healthy individual:
A) RDA B) REA C) RAD D) RAE E) None

www.brex.us 282
MICROBIOLOGY & PUBLIC HEALTH

B 81. A type of malnutrition associated with inadequate mastication, digestion, absorption, transport and
excretion of nutrients:
A) Primary malnutrition D) Overnutrition
B) Secondary malnutrition E) None
C) Undernutrition

A 82. Deficiency of thiamine leads to:


A) Beri-beri B) Pellagra C) Stomatitis D) A and B E) B and C

C 83. A short-term expression of alcohol toxicity:


A) Cirrhosis B) Alcoholism C) Hang-over D) A and B E) B and C

C 84. Vitamin B deficiency caused by alcoholism produces a neurological disorder called:


A) Steven Johnson’s Syndrome D) A and B
B) Down Syndrome E) None
C) Wernicke-Korsakoff Syndrome

B 85. Regarding the nature of medicinally important viruses, which one of the following statements is least
accurate?
A) Poliovirus is a non-enveloped virus with RNA as its genome.
B) Epstein-Barr virus is a non-enveloped virus with RNA as its genome.
C) Hepatitis B is an enveloped virus with RNA as its genome.
D) Influenza virus is an enveloped virus with RNA as its genome.

D 86. The following statements regarding the capsules of bacteria are correct, except:
A) Most bacterial capsules are polysaccharides and serve to protect the bacteria by inhibiting
phagocytosis.
B) Bacterial capsules can vary antigenically, and as a result some bacteria have many serologic types.
C) Bacterial capsules can be purified and used in vaccines against certain bacteria, example the
Pneumococcus.
D) Most gram-positive bacteria have capsules, whereas gram-negative bacteria rarely do.

C 87. The following statements regarding Coccidiodes immitis are correct, except:
A) It is a dimorphic fungus that grows as a mold in the soil and as spherules in the body.
B) Infection usually results from the inhalation of asexual spores (anthroconida), hence the primary site
of infection is the lungs.
C) When cultured in the laboratory, the organism forms budding yeasts.
D) The most important host defense against this organism is cell-mediated immunity.

A 88. The following statements regarding bacterial exotoxins are correct, except:
A) They are integral parts of the cell wall.
B) They are produced by both Staphylococcus aureus and Escherichia coli.
C) They are polypeptides consisting of two functional regions, the one that binds to cell receptors and
one that ha the toxic activity.
D) Treatment of some exotoxins with formaldehyde yields a toxoid, which is used as the immunogen in
certain vaccines.

D 89. The following statements regarding the C3 component of the complement cascade are correct, except:
A) It is involved in both the classic and the alternative pathways.
B) Its C3a fragment can cause anaphylaxis by releasing histamines from mast cells.
C) Its C3b fragment binds to both IgG and surface receptors on neutrophils.
D) Its C3b fragment is part of the complex that causes lysis of gram-negative bacteria such as Neiserria.

C 90. Regarding the prevention of bacterial diseases by vaccines, which one of the following is least accurate?
A) Tetanus toxoid is produced by treating tetanus toxin with formalin, which inactivates its ability to cause
disease but leaves its antigenicity intact.
B) Diphtheria vaccine contains diphtheria toxoid and produces few side effects when given to children.
C) Both the pertussis vaccine and Haemophilus influenza vaccine contain inactivated whole bacteria and
produce significant side effects in children.
D) The pneumococcal vaccine contains the capsular polysaccharide of many serotypes and is
recommended primarily for older people.

D 91. Several viruses infect the intestinal tract as their initial site of infection. Which one of the following is least
likely to do this?
A) Hepatitis A virus B) Poliovirus C) Rotavirus D) Mumps virus

C 92. Penicillin is a very effective antibacterial drug but their use is limited by allergic reactions. In these
allergies, Penicillin acts as a hapten. Which of the following is the most accurate?

A) Penicillin is a T-dependent antigens, which bind to receptors on B cells and stimulate an antibody
response.
B) Penicillin interacts with T cell receptors on CD4-positive T cells and activates them.
www.brex.us 283
MICROBIOLOGY & PUBLIC HEALTH

C) Penicillin binds to carrier proteins, then interacts with the B cell receptor and carrier proteins. The
carrier protein epitope is presented to the helper-T cell.
D) Penicillin interacts with the early complements (C1, C4, C2 and C3) to release inflammatory
mediators.

Choices for numbers 93 – 97:


A) Staphylococcus aureus D) Staphylococcus epidermidis
B) Streptococcus pyogenes E) Streptococcus agalactiae
C) Streptococcus pneumoniae

B 93. The causative agent of suppurative diseases like pharyngitis and cellulites and nonsuppurative diseases
like rheumatic fever and acute glomerulonephritis.

A 94. The causative agent of food poisoning, TSS and skin boils.

D 95. Norma flora of the skin but may cause “stitch abscess” and sepsis.

B 96. A group B Streptococcus that causes neonatal meningitis and sepsis.

C 97. The causative agent of pneumonia and meningitis in adults, and otitis media and sinusitis in children.

Choices for numbers 98 – 102:


A) Meningococcus D) Clostridium tetani
B) Gonococcus E) Listeria monocytogenes
C) Bacillus antracis

B 98. A gram negative, kidney bean shaped diplococci causing gonorrhea.

A 99. A gram negative, kidney bean shaped diplococci, oxidase positive with large polysaccharide capsule
referred to as glycocalyx.

D 100. Anaerobic, gram positive with terminal spore affecting neutrotransmitters leading to excitatory neurons
that are unopposed and extreme muscle spasm.

E 101. Aerobic, non-spore forming organism that exhibits tumbling motility and capable of causing meningitis
and sepsis in newborn and immunocompromised patients.

C 102. Large, spore-forming rod, whose capsule is composed of poly-D-glutamate and may cause Wool Sorter’s
disease.

Choices for numbers 103 – 106:


A) Clostridium perfringens D) Corynebacterium diphtheriae
B) Clostridium tetani E) Clostridium dificile
C) Clostridium botulinum

B 103. Characterized physiologically by a “sardonic smile.”

A 104. The causative agent of gas gangrene.

D 105. In stained smear, it is usually seen in Chinese letter arrangement having metachromatic granules and
may infect the respiratory system.

E 106. Anaerobic, gram positive, spore-forming rods, which is a normal flora of the intestine. It has enterotoxin
that causes watery diarrhea and may lead to pseudomembranous colitis.

C 107. The causative agent of food poisoning from canned good foods.

Choices for numbers 108 – 112:


A) Escherichia coli D) Vibrio cholerae
B) Salmonella typhi E) Camphylobacter jejuni
C) Shigella dysenteriae

D 108. A halophilic coma shaped, gram negative bacteria.

E 109. A gram negative, coma shaped microaerophilic rod that causes enterocolitis and can be cultured in
Skirrow’s agar.

C 110. Facultative gram negative rods, non-lactose fermenting and may cause enterocolitis (dysentery).
A 111. Urinary tract infection (UTI), sepsis neonatal meningitis and “travelers diarrhea” are the most common
diseases caused by this gram negative, non-lactose fermenting bacilli.

B 112. Ceftriaxone is the most effective drug used to treat this facultative, non-lactose fermenting, gram
negative rod which is capable of producing H2S.
www.brex.us 284
MICROBIOLOGY & PUBLIC HEALTH

Choices for numbers 113 – 117:


A) Mycobacterium leprae D) Treponema pallidum
B) Mycobactrium tuberculosis E) Leptospira interrogans
C) Borrelia burgdorferi

D 113. Penicillin is effective for this spirochete organism causing syphilis.

E 114. Spirochete, which is the causative agent of leptospirosis.

C 115. Spirochete, which is the causative agent of Lyme disease and can be treated by Doxycycline for early
stage and Pen G for late stages.

A 116. An acid-fast bacillus that have mycolic acid in its cell wall, which can be seen in a palisade arrangement.

B 117. An acid-fast, catalase negative bacillus that can be grown in Lowenstein-Jensen medium and is capable
of producing Niacin. It can be treated with Rifampicin, Pyrazinamide and INH.

Choices for numbers 118 – 122:


A) Herpes Simplex virus Type 1 D) Cytomegalovirus
B) Varicella-Zoster virus E) Epstein-Barr virus
C) Herpes simplex virus Type 2

D 118. Gangcyclovir is beneficial in treating pneumonia and retinitis, while Acyclovir is ineffective.

E 119. The causative agent of Infectious Mononucleosis (IM) and is associated with Burkitt’s lymphoma in East
African children. No drug is effective to treat the disease.

C 120. The causative agent of herpes genitalis, which can then be treated with Acyclovir.

A 121. The causative agent of herpes labialis (fever blisters or cold sores), keratitis and encephalitis.

B 122. The causative agent of Varicella (chicken pox) in children and Zoster (shingles) in adults.

D 123. Which one of the following statements is the most accurate comparison of human, bacterial and fungal
cells?
A) Human cells undergo mitosis, whereas neither bacteria nor fungi do.
B) Human and fungal cells have similar cell wall, in contrast to bacteria whose cell wall contains
peptidoglycan.
C) Human and bacterial cells have plasmids, whereas fungal cells do not have.
D) Human and fungal cells have similar ribosomes, whereas bacterial ribosomes are different.

C 124. The following statements concerning endotoxins are correct, except:


A) They are less potent (ie., less active on weight basis) than exotoxins.
B) They are more heat stable than exotoxins.
C) They bind to specific cell receptors, whereas exotoxins do not.
D) They are part of the bacterial cell wall, whereas exotoxins are not.

B 125. The main host defense against bacterial exotoxins is:


A) Activated macrophages secreting proteases
B) IgG and IgM antibodies
C) Helper-T cells
D) Modulation of host cell receptors in response to the toxin

D 126. The following events involve recombination of DNA, except:


A) Transduction of a chromosomal gene
B) Transposition of a mobile genetic element
C) Integration of a temperate bacteriophage
D) Conjugation, such as the transfer of a R (resistance) factor

B 127. The following statements about the normal flora are correct, except:
A) The most common organism found on the skin is Staphylococcus epidermidis.
B) Escherichia coli is a prominent member of the normal flora of the throat.
C) Colon is the major site where Bacteroides fragilis can be found.
D) Nose is one of the most common sites where Staphylococcus aureus can be found.

C 128. Which of the following statements is the most important function of antibody in host defenses against
bacteria?
A) Activation of the lysozyme that degrades the cell wall
B) Acceleration of proteolysis of exotoxins
C) Facilitation of phagocytosis
D) Inhibition of bacterial protein synthesis

www.brex.us 285
MICROBIOLOGY & PUBLIC HEALTH

D 129. Which of the following best describes the mode of action of endotoxin?
A) Degrades lecithin in cell membranes
B) Inactivates elongation factor-2
C) Blocks release of acetylcholine
D) Causes the release of necrosis factor

D 130. The identification of bacteria by serologic test is based on the presence of specific antigens. Which of the
following bacterial components is least likely to contain useful antigens?
A) Capsule B) Flagella C) Cell wall D) Ribosomes

Choices for numbers 131 – 135:


A) Diphtheria toxin D) Toxic shock syndrome toxin
B) Tetanus toxin E) Cholera toxin
C) Botulinum toxin

C 131. Causes paralysis by blocking release of acetylcholine

A 132. Inhibits protein synthesis by blocking elongation factor-2

D 133. Stimulates T cells to produce cytokines

E 134. Stimulates the production of cyclic AMP by adding ADP-ribose to a G protein

B 135. Inhibits the release of inhibitory neurotransmitters causing muscle spasms

B 136. An outbreak of sepsis causes by Staphylococcus aureus has occured in the newborn nursery. You are
called to investigate. According to your knowledge of the norma flora , what is the most likely source of
the bacteria?
A) Colon B) Nose C) Throat D) Vagina

A 137. The following organisms are recognized causes of diarrhea, except:


A) Clostridium perfringens B) Streptococcus fecalis C) Escherichia coli D) Vibrio cholerae

C 138. A patient has subacute bacterial endocarditis, which is caused by a member of the viridans group of
Streptococcus. Which one of the following sites is most likely to be the source of the organism?
A) Skin B) Colon C) Oropharynx D) Urethra

C 139. The coagulase test, wherein the bacteria causes plasma to clot is used to distinguish:
A) Streptococcus pyogenes from Streptococcus faecalis
B) Streptococcus pyogenes from Staphylococcus aureus
C) Staphylococcus aureus from Staphylococcus epidermidis
D) Staphylococcus epidermidis from Neisseria meningitidis

C 140. Five hours after eating fried rice at a restaurant, you and your friends developed nausea, vomiting and
diarrhea. Which of the following organisms is most likely to be the causative agent?
A) Clostridium perfringens C) Bacillus cereus
B) Enterotoxigenic Escherichia coli D) Salmonella typhi

C 141. Which of the following zoonotic diseases has no arthropod vector?


A) Plague B) Lyme disease C) Brucellosis D) Epidemic typhus

B 142. Which of the following organisms principally infects vascular endothelial cells?
A) Salmonella typhi B) Rickettsia typhi C) Haemophilus influenzae D) Coxiella burnetii

D 143. The following statements concerning Chlamydia are correct, except:


A) Chlamydia is a strict intracellular parasite because it cannot synthesize sufficient ATP.
B) Chlamydia possesses both DNA and RNA and is bounded by a cell wall.
C) Chlamydia trachomatis has multiple serotypes, whereas C. psittaci has only one serotype.
D) Most Chlamydias are transmitted by arthropods.

D 144. A 55-year old man develops dysuria and hematuria. A gram stain of urine sample shows gram-negative
rods. Culture of the urine on EMB agar reveals non-lactose fermenting colonies without evidence of
swarming motility. Which of the following organisms is most likely to be the causative agent of his urinary
tract infection?

A) Streptococcus faecalis B) Pseudomonas aeruginosa


C) Proteus vulgaris D) Escherichia coli

B 145. Acute glomerulonephritis is a nonsuppurative complication that follows infection by which of the following
organisms?
A) Streptococcus faecalis B) Streptococcus pyogenes
C) Streptococcus pneumoniae D) Streptococcus agalactiae

www.brex.us 286
MICROBIOLOGY & PUBLIC HEALTH

C 146. Which of the following organisms is most likely to be the cause of pneumonia in an immunocompetent
patient?
A) Nocardia asteroides B) Serratia marcescens
C) Mycoplasma pneumoniae D) Legionella pneumophila

A 147. Which of the following forms of immunity to viruses would be least likely to be lifelong?
A) Passive immunity B) Passive-active immunity
C) Active immunity D) Cell-mediated immunity

D 148. The following statements concerning Interferon are correct, except:


A) Interferon inhibits the growth of both DNA and RNA viruses.
B) Interferon is induced by double-stranded DNA.
C) Interferon of one species acts more effectively in the cells of that species.
D) Interferon acts by preventing viruses from entering the cell.

C 149. Viruses are obligate intracellular parasites. The following statements concerning this fact are correct,
except:
A) Viruses cannot generate energy outside the cell.
B) Viruses cannot synthesize proteins outside the cell.
C) Viruses must degrade host cell DNA in order to obtain nucleotides.
D) Enveloped viruses require host cell membranes to obtain their envelopes.

A 150. The following statements about lysogeny are correct, except:


A) Viruses replicate independently of bacterial genes.
B) Viral genes responsible for lysis are repressed.
C) Viral DNA is integrated into bacterial DNA.
D) Some lysogenic bactriophage encode toxins that cause human diseases.

B 151. The following viruses possess an outer envelope of lipoprotein, except:


A) Varicell-zoster virus B) Papillomavirus C) Influenza virus D) HIV virus

B 152. The following viruses possess RNA polymerase in the virion, except:
A) Hepatitis A virus B) Smallpox virus C) Mumps virus D) Rotavirus

A 153. The following viruses possess double-stranded DNA as its genome, except:
A) Coxsackie virus B) Herpes simplex virus C) Rotavirus D) Adenovirus

B 154. Which of the following statements best describe a viriod?


A) It is a defective virus that is missing the DNA coding for the matrix protein.
B) It consists of RNA without a protein or lipoprotein outer coat.
C) It causes tumor in experimental animals.
D) It requires RNA polymerase in the particle for replication to occur.

B 155. The following statements about measles virus and rubella virus are correct, except:
A) They are enveloped RNA viruses.
B) Their virions contain RNA polymerase.
C) They each have a single antigenic type.
D) They are transmitted by respiratory aerosol.

D 156. The following statements about influenza virus and rabies virus are correct, except:
A) They are enveloped RNA viruses.
B) Their virions contain RNA polymerase.
C) Vaccines containing killed organisms are available for both viruses.
D) They each have a single antigenic type.

C 157. The following statements about poliovirus and rhinovirus are correct, except:
A) They are non-enveloped RNA viruses.
B) They each have multiple antigenic types.
C) Their virions contain RNA polymerase.
D) They do not integrate their genome into the DNA of the host cell.
Choices for numbers 158 – 161:
A) DNA enveloped virus D) RNA non-enveloped virus
B) DNA non-enveloped virus E) Viriod
C) RNA enveloped virus
A 158. Herpes simplex virus

C 159. Human Tcell leukemia virus

B 160. Human papillomavirus

D 161. Rotavirus

B 162. The following pathogens are likely to establish chronic or latent infections, except:
www.brex.us 287
MICROBIOLOGY & PUBLIC HEALTH

A) Cytomegalovirus B) Hepatitis A virus C) Hepatitis B virus D) Herpes simples virus

D 163. Which of the following strategies is most likely to induce lasting intestinal mucosal immunity to
poliovirus?
A) Parenteral (intramuscular) administration of inactivated vaccine
B) Oral administration of poliovirus immunoglobulin
C) Parenteral administration of live vaccine
D) Oral administration of live vaccine

C 164. The following clinical syndromes are associated with infection by picornavirus, except:
A) Myocarditis/Pericarditis B) Hepatitis C) Mononucleosis D) Meningitis

A 165. The following statements concerning human rabies vaccine are correct, except:
A) The vaccine contains live, attenuated rabies virus.
B) If the patient was bitten by a wild animal such as skunk, the rabies vaccine must be given.
C) When the vaccine is used for post-exposure prophylaxis, rabies immune globulin must also be given.
D) The virus in the vaccine is grown in human cell cultures thus decreasing the risk of allergic
encephalomyelitis.

C 166. The following statements about Hepatitis A are correct, except:


A) The initial site of viral replication is the gastrointestinal tract.
B) Hepatitis A virus commonly causes asymptomatic infection in children.
C) Isolating the virus in a cell culture usually makes diagnosis of hepatitis A.
D) Gamma globulin is used to prevent the disease in exposed persons.

Choices for numbers 167 – 171:


A) Hepatitis C virus D) Dengue virus
B) Cytomegalovirus E) St. Louis encephalitis virus
C) Human papilloma virus

C 167. It is implicated as the cause of carcinoma of the cervix.

E 168. Wild birds are important reservoirs of this virus.

B 169. It is an important cause of pneumonia in immunocompromised patients.

A 170. Donated blood containing antibody to this RNA virus ahould not be used for transfusion.

D 171. It causes hemorrhagic fever that can be life threatening.

D 172. Bacteria lack all of the following organelles, except:


A) Mitochondria B) Nucleus C) Mitotic apparatus D) DNA E) Nuclear membrane

D 173. The following are true about prokaryotes, except:


A) They are relatively small.
B) Their genes are dedicated to essential functions only.
C) They lack autonomous organelles.
D) Their genetic materials are enclosed in a nuclear membrane.

A 174. Unicellular prokaryotic organism that divides by binary fission:


A) Bacteria B) Fungi C) Protozoa D) Helminths

B 175. Phylogenetic classifications are based on:


A) Shared morphologic attributes B) Evolutionary relationships
C) Shape D) Unusual growth characteristics

E 176. The peptidoglycan of gram-negative bacteria probably is:


A) 20 layers thick D) 80 layers thick
B) 40 layers thick E) 1 layer thick
C) 10 layers thick

C 177. Porins are:


A) Cytoplasmic membrane proteins B) Periplasmic proteins
C) Outer membrane proteins D) Inclusion bodies

B 178. Gram-negative bacterial cell wall contains the following components, except:
A) Lipoproteins B) Lipotechoic acid C) Lipopolysacchatide D) Phospholipid

C 179. This is a saclike invagination of the cytoplasmic membrane that is associated with the DNA of bacterial
cells. It functions as the origin of the transverse septum that divides the cell and the binding site of the
DNA.
A) Nucleoid B) Plasmids C) Mesosomes D) Transposons

www.brex.us 288
MICROBIOLOGY & PUBLIC HEALTH

A 180. The bacterial organelle that is used for motility:


A) Flagella B) Pili C) Cilia D) Pseudopods

B 181. An intracytoplasmic granule may contain:


A) Nucleic cids B) Glycogen C) Steroids D) Spindle fibers

C 182. The substance responsible for the heat resistance of endospores:


A) Polymerized d-glutamic acid B) Flagellin C) Dipicolinic acid D) Techoic acid

B 183. This mediates the attachment of bacteria to specific receptors on human cell surfaces, which is the
necessary step in the initiation of infection for some organisms:
A) Flagella B) Fimbriae C) Capsule D) Endospores

B 184. This mediates the firm adherence of bacteria to various structures such as skin, heart valves, teeth and
even catheters:
A) Capsule B) Slime layer C) Pili D) Flagella

D 185. The following are true for endospores, except:


A) Highly resistant to disinfectants
B) Can survive for many years, especially in soil
C) Produced under condition of nutritional deprivation
o
D) Killed by heating at 100 C

A 186. A rod-like bacterium that grows optimally at mammalian body temperature and is killed by the presence
of oxygen is best described as:
A) Mesophilic, obligate anaerobic bacilli
B) Mesophilic, obligate aerobic bacilli
C) Psychrophilic, facultative anaerobic streptobacillus
D) Psychrophilic, obligate anaerobic bacilli

A 187. Bacteria that derive their energy from oxidation of organic molecules are known as:
A) Heterotrophs B) Autotrophs C) Chemoautotrophs D) Photoautotrophs

C 188. The stage in the growth cycle of bacteria that reflects the period during which vigorous metabolic activity
occurs but cells do not divide is the:
A) Stationary phase B) Log phase C) Lag phase D) Death phase

C 189. The time required for a cell to divide or a population to double is known as:
A) Thermal death time B) Growth rate constant C) Generation time D) Thermal death point

C 190. Which of the following organisms is a predatory bacterium?


A) Salmonella thypinurium C) Bdellovibrio bacteriovirus
B) Pseudomonas aeruginosa D) Corynebacterium diphtheriae

D 191. Specific nutrients transverse the cytoplasmic membrane of bacteria by:


A) Activated transport B) Facilitated diffusion
C) Carrier-mediated transport D) Any of the above

B 192. Bacteria that ferment substrates to single end products are called:
A) Heterofermenters B) Homofermenters
C) Glyoxalate shunt fermenters D) Oxidative fermenters

B 193. The primary natural nutrients of bacteria are:


A) Carbohydrates/Nucleic acids/Steroids C) Nucleic acids/Chitin/Peptidoglycans
B) Carbohydrates/Amino acids/Lipids D) Lipids/Polysaccharides/Lactoferrin

C 194. Which of the following statements is true about drug-resistant plasmids?


A) Found only in gram-negative bacteria
B) Usually confer resistance to a single antibiotic
C) May be divided into two distinct genetic components: the RTF and r determinant
D) Two of the above

B 195. Which of the following statements is true about bacterial conjugation?


A) It requires the F factor
B) It is controlled by F plasmid and sex pili
C) It is mediated by a bacterial virus
D) It is found only in gram-negative bacteria

C 196. It is the relationship between the host and the normal microbiota:
A) Disease B) Infection C) Symbiosis D) Parasitism

C 197. It refers to the invasion or colonization of the body by pathogenic bacteria:


A) Disease B) Infestation C) Infection D) Syndromes
www.brex.us 289
MICROBIOLOGY & PUBLIC HEALTH

A 198. Influenza develops rapidly and lasts only for a short time. This is described as:
A) Acute B) Subacute C) Chronic D) Latent

B 199. This period is characterized by mild aches, malaise and it follows the period of incubation of the disease:
A) Decline phase B) Prodormal stage C) Illness stage D) Convalescence stage

C 200. The number of cases of Filipinos with AIDS has increased tremendously over the past few years. This is
described as:
A) Endemic B) Epidemic C) Pandemic D) Acute

B 201. It is an indication of the ability of a bacterium to produce pathologic changes or disease in the host:
A) Virulence B) Pathogenicity C) Attenuation D) In vitro growth rates

A 202. The two major components of virulence include:


A) Invasiveness and toxigenicity C) Adherence factors and antiphagocytic mechanisms
B) Intracellular and extracellular survival D) Capsules and spreading factors

B 203. The bacterial toxin that attaches to the presynaptic terminals of cholinergic nerves, where it blocks the
release of acetycholine, is produced by:
A) Bacillus anthracis C) Clostridium tetani
B) Clostridium botulinum D) Corynebacterium diphtheriae

D 204. The bacterial toxin that causes cessation of mammalian protein synthesis by inactivating EF-2 of
eukaryotic cells is produced by:
A) Bacillus anthracis C) Clostridium tetani
B) Clostridium botulinum D) Corynebacterium diphtheriae

A 205. The bacterium that produces a toxin that activates adenylate cyclase, resulting in accumulation of cyclic
AMP in the epithelial cells of the mucosal lining, is:
A) Escherichia coli C) Streptococcus pneumoniae
B) Pseudomonas aeruginosa D) Staphylococcus aureus

D 206. The following statements are true about endotoxins, except:


A) They are also known as lipopolysaccharides.
B) They are integral part of the cell walls of gram-negative bacteria.
C) They are normally not as toxic as bacterial exotoxins.
D) Their toxicity can be destroyed by autoclaving.

C 207. The following structures normally contain indigenous microbial flora, except:
A) Teeth B) Urethra C) Bronchi D) Skin

D 208. The region of the body that contains the largest population of microbial flora:
A) Skin B) Colon C) Vagina D) Mouth

B 209. Vancomycin is produced naturally from:


A) Strep. nodosus B) Strep. orientalis C) Strep. griseus D) Strep. orchidaceous

C 210. Polymixin is produced from:


A) Bacillus subtilis B) Bacillus cereus C) Bacillus polymyxa D) Bacillus anthracis

D 211. The following antibiotics interfere with the synthesis of proteins in microorganisms, except:
A) Aminoglycosides B) Tetracyclines C) Lincomycin D) Cephalosporins

C 212. Certain bacterial strains are resistant to the bactericidal activity of the Penicillins and the Cephalosporins,
although the growth of the organism is inhibited. This phenomenon (tolerance) is related to a defect or a
deficiency of:
A) Plasmid that codes for penicillinase C) Cellular growth rates
B) Autolytic enzymes D) Certain cytochromes

B 213. Nystatin is produced naturally from:


A) Strep. nodosus B) Strep.noursei C) Strep. griseus D) Strep. natalensis

D 214. This antibiotic has been found to be toxic, so its use has been limited only for the treatment of TB:
A) Penicillin B) Bacitracin C) Vancomycin D) Cycloserine

B 215. Aplastic anemia is a rare but usually fatal side effect associated with the use of this antibiotic:
A) Gentamicin B) Chloramphenicol C) Bacitracin D) Polymixin

B 216. It refers to the killing of many, but not all microorganisms:


A) Sterilization B) Disinfection C) Antisepsis D) Asepsis

D 217. It is the most effective skin antiseptic used in medical practice:


www.brex.us 290
MICROBIOLOGY & PUBLIC HEALTH

A) Chlorine B) Merthiolate C) Hydrogen peroxide D) Iodine

A 218. This chemical agent is used extensively in hospitals for the sterilization of heat sensitive materials. It
kills by alkylating both proteins and nucleic acids:
A) Ethylene oxide B) Formaldehyde C) Acids and alkalis D) Hydrogen peroxide

B 219. It is the method of choice in sterilizing heat sensitive solutions:


A) Autoclave B) Filtration C) Radiation D) Pasteurization

D 220. This disinfectant is useful in preventing gonococcal ophthalmia neonatorum among infants whose
patients have Gonorrhea:
A) Merthiolate B) Mercurochrome C) Silver sulfadiazine D) Silver nitrate

D 221. The following are milk-borne pathogens, except:


A) Mycobaterium bovis B) Salmonella C) Streptococcus D) Clostridium

D 222. Which is the best method to sterilize Penicillin solution?


A) Autoclaving B) Dry heat (oven) C) Ethylene oxide D) Filtration

C 223. The DTP vaccine is composed of which of the following:


A) Diphtheria toxin, tetanus toxin, heat-killed Bordetella pertussis
B) Diphtheria toxoid, tetanus toxoid, heat-killed Pseudomonas aeruginosa
C) Diphtheria toxoid, tetanus toxoid, heat-killed Bordetella pertussis
D) Diphtheria toxin, tetanus toxin, heat-killed Pseudomonas aeruginosa

B 224. The capsule of Bacillus anthracis is composed of:


A) L-glutamic acid B) D-glutamic acid C) Polysaccharide D) Nucleic acid

D 225. Bacillus anthracis possesses all of the following characteristic, except:


A) It is a large, gram-positive rod
B) It is a spore-former.
C) It produces a potent exotoxin composed of three antigenically distinct thermolabile proteins.
D) It can live inside phagocytic cells.

A 226. Listeria monocytogenes has all of the following characteristics, except:


A) It is a spore-former. C) It can live inside phagocytic cells.
B) It is a short, gram-positive rod D) It is actively motile.

A 227. The most important anti-phagocytic structure of the Group A streptococci:


A) M protein B) Group A carbohydrate C) Hyaluronic acid capsule D) Protein A

C 228. The single differentiation test between Staphylococcus epidermidis and Staphylococcus aureus:
A) Gram staining B) Catalase test C) Coagulase test D) Hemolysis test

D 229. An example of a streptococcal disease sequela:


A) Scarlet fever B) Pharyngitis C) Puerperal sepsis D) Rheumatic fever

C 230. The group A streptococci serotype most commonly associated with acute glomerulonephritis:
A) 2 B) 7 C) 12 D) 19

C 231. The Streptococcus most commonly associated with subacute bacterial endocarditis:
A) Streptococcus pyogenes C) Viridans streptococci
B) Streptococcus agalactiae D) Streptococcus pneumoniae

C 232. Which of the following does not influence the virulence of Neisseria gonorrhea?
A) Polysaccharide capsule C) Genus-specific protein
B) Lipopolysaccharide endotoxin D) Production of IgA protease

D 233. Neisseria are all:


A) Gram positive B) Anaerobes C) Coagulase positive D) Oxidase positive

D 234. In the laboratory, Neisseria meningitidis may be distinguished from Neisseria gonorrhea by:
A) Its characteristic morphology and gram stain
B) Its unique requirement for CO2 and ability to grow in Thayer-Martin medium
C) The presence of pili
D) Its ability to ferment particular carbohydrate

C 235. The most important mechanism of transmission of Pasteurella multicoda from animals to humans:
A) Blood-sucking arthropods D) Contact with contaminated urine
B) Contact with contaminated feces E) Handling of infected tissues
C) Animal bites and scratches

B 236. Which of the following statements is true about whooping cough?


www.brex.us 291
MICROBIOLOGY & PUBLIC HEALTH

A) Infants under the age of 3 months are not susceptible.


B) The causative organism adheres to ciliated epithelial cells of the upper respiratory tract.
C) Immunity is conferred by killer T cells that recognize cell wall M antigen.
D) All of the above

E 237. Tularemia may occur in:


A) Ulderoglandular form B) Oculoglandular form C) Pulmonary form D) All of the above

D 238. The intestinal tract is the only natural habitat of this lactose fermenting, facultative anaerobic,
nosocomial pathogen:
A) Pseudomonas aeruginosa C) Klebsiella pneumoniae
B) Proteus vulgaris D) Escherichia coli

A 239. A gram negative, non-motile rod that produces a large mucoid colony on agar medium was isolated
from a chronically ill patient who died of pneumonia. This organism probably belongs to the genus:
A) Klebsiella B) Pseudomonas C) Proteus D) Serratia

A 240. A gram-negative rod was isolated from the urine of a patient suspected of having a UTI. The organism
is lactose negative, urease positive and motile. It most likely belongs to the genus:
A) Proteus B) Escherichia C) Serratia D) Klebsiella

C 241. Leprosy may manifest itself as:


A) Lepromatous leprosy B) Tuberculoid leprosy C) A and B D) B and C

D 242. Secondary syphilis is characterized by all of the following, except:


A) Cutaneous lesions C) Onset is 4 weeks or more after chancre
B) Mucous membrane lesions D) Absence of spirochetes in the lesions

D 243. The diagnosis of early syphilis is best made by:


A) Inoculating guinea pigs with lesion material and examining blood smears 3 days later
B) Culture on chocolate agar and incubated with 10% carbon dioxide
C) The VDRL test
D) Dark-field microscopy of lesion scrapings if a chancre is present

B 244. The smallest known free-living microorganism is:


A) Viruses B) Mycoplasma species C) Rickettsia D) Chlamydiae

B 245. The growing stage of a protozoan parasite is:


A) Sporozoite B) Trophozoite C) Merozoite D) Cysts

D 246. A class in which only parasitic forms occur:


A) Sarcodina B) Mastigophora C) Ciliata D) Sporozoa

D 247. In which of the following the eggs of parasitic worms can be found:
A) Stool B) Urine C) Sputum D) All of the above E) None of the above

E 248. Which of the following techniques is used in quantitative estimations of infections with parasitic worms?
A) Formalin-ether B) Kato thick smear C) Stool dilution D) A and B E) B and C

B 249. Blastocyctis hominis is a:


A) Zooparasite B) Yeast resembling a protozoan cyst
C) New name for Enatamoeba coli D) Bacteria

C 250. The findings of ingested red blood cells in a trophozoite practically identifies:
A) Entamoeba coli C) Entamoeba histolytica E) Dientamoeba fragilis
B) Endolimax nana D) Iodamoeba butschlii

C 251. Actively motile trophozoite with directional motility and contains ingested red blood cells:
A) Entamoeba coli C) Entamoeba histolytica E) Dientamoeba fragilis
B) Endolimax nana D) Iodamoeba butschlii

A 252. Trophozoite with nucleus seen without difficulty in fresh, unstained preparation:
A) Entamoeba coli C) Entamoeba histolytica E) Dientamoeba fragilis
B) Endolimax nana D) Iodamoeba butschlii

C 253. Cysts of this organism contains 1-4 nuclei:


A) Entamoeba coli C) Entamoeba histolytica E) Dientamoeba fragilis
B) Endolimax nana D) Iodamoeba butschlii

C 254. The chromatoid bodies of this organism appear as cigar-shaped or sausage-shaped:


A) Entamoeba coli C) Entamoeba histolytica E) Dientamoeba fragilis
B) Endolimax nana D) Iodamoeba butschlii
www.brex.us 292
MICROBIOLOGY & PUBLIC HEALTH

D 255. Cysts of this organism are usually without glycogen vacuoles (Iodine cysts):
A) Entamoeba coli C) Entamoeba histolytica E) Dientamoeba fragilis
B) Endolimax nana D) Iodamoeba butschlii

A 256. In clinical cases of malaria, the most favorable time to find parasites in the blood is:
A) The period just before a paroxysm
B) At the beginning of a paroxysm
C) During the late paroxysmal stage
D) The period just following a paroxysm

B 257. Definitive host of human malarial parasites:


A) Any domestic animal B) Anopheles mosquito C) Man D) B and C

A 258. The sexual cycle is the:


A) Sporogony B) Schizogony C) Either B or C D) Neither B nor C

A 259. Eosinophilic stippling of erythrocytes (Schauffer.s dots) is often seen in infections with:
A) Plasmodium vivax B) Plasmodium malariae
C) Plasmodium falciparum D) Plasmodium ovale

C 260. As a rule, only rings and crescent-shaped gametocytes are observed in peripheral blood, which
includes:
A) Plasmodium vivax B) Plasmodium malariae
C) Plasmodium falciparum D) Plasmodium ovale

A 261. The malarial parasite that enlarges red blood cells:


A) Plasmodium vivax B) Plasmodium malariae
C) Plasmodium falciparum D) Plasmodium ovale

B 262. Man may acquire Toxoplasma from:


A) Mice B) Cats C) Dogs D) Pigs

C 263. Toxoplasmosis can be diagnosed by:


A) Stool examination B) Thin blood film
C) Flourescent antibody tests D) Intradermal test

B 264. Toxoplasma is probably transmitted to man by:


A) Bite of the vector insect
B) Ingestion of a resistant oocyst
C) In the body of the parasitic nematode
D) Ingestion of infected food

A 265. The posterior part of this parasite has a twisted appearance:


A) Chilomastix mesnili B) Giardia lamblia
C) Trichomonas hominis D) Retortamonas

C 266. An axostyle is present in this organism:


A) Plasmodium vivax B) Plasmodium malariae
C) Plasmodium falciparum D) Plasmodium ovale

B 267. Trophozoite that resembles “old man’s eyeglasses”:


A) Plasmodium vivax B) Plasmodium malariae
C) Plasmodium falciparum D) Plasmodium ovale

A 268. This organism has lemon-shaped cysts:

A) Plasmodium vivax B) Plasmodium malariae


C) Plasmodium falciparum D) Plasmodium ovale

A 269. This causes visceral leishmaniasis:


A) Leishmania donovani C) Leishmania braziliensis
B) Leishmania tropica D) Trypanosoma cruzi

C 270. The reservoir host of Balantidium coli infection:


A) Dog B) Cat C) Pig D) Cattle

B 271. A viviparous organism:


A) Ascaris lumbricoides B) Trichenella spiralis C) Hookworm D) Pinworm

B 272. Which of the following neither lay eggs nor deposits larvae in the intestinal canal?
A) Hookworm B) Pinworm C) Trichinella spiralis D) Trichuris trichiura

www.brex.us 293
MICROBIOLOGY & PUBLIC HEALTH

B 273. “Old world” hookworm:


A) Necator americanus B) Ancylostoma duodenale
C) Ancylostoma caninum D) Toxocara canis

C 274. This organism causes tropical elephantiasis:


A) Loa loa B) Onchocerca volvulus C) Wucheria bancrofti D) Dracunculus medinensis

C 275. The presence of mammillated eggs or the large adult worm in the stool identifies this organism:
A) Trichuris trichiura B) Enterobius vermicularis
C) Ascaris lumbricoides D) Strongyloides stercoralis

B 276. The adult form of this organism has a whip-like anterior portion:
A) Enterobius vermicularis B) Trichuris trichiura C) Ascaris lumbricoides D) Trichinella spiralis

A 277. The eggs of this organism are most likely to be found through anal swabs or cellophane tape:
A) Enterobius vermicularis B) Trichuris trichiura C) Ascaris lumbricoides D) Trichinella spiralis

C 278. Visceral larva migrans is acquired by ingesting eggs of a nematode parasite of:
A) Pig B) Monkey C) Dog D) Cat

E 279. Filariform larva that can penetrate through the skin:


A) Enterobius vermicularis D) A and B
B) Strongyloides stercoralis E) B and C
C) Hookworm

E 280. Finding the rhabditiform larva in the stool identifies this organism:
A) Hookworm D) Ascaris lumbricoides
B) Whipworm E) Strongyloides stercoralis
C) Pinworm

B 281. Eggs in the 2-8 cell stage of development often appear in the stool in infections with
A) Whipworm B) Hookworm C) Pinworm D) Large intestinal roundworm

C 282. The dog tapeworm that can be accidentally transmitted to humans:


A) Hymenolepsis nana B) Hymenolepsis diminuta C) Dipylidium caninum D) Taenia solium

C 283. The fish tapeworm that can be acquired by eating insufficiently cooked, infected fish:
A) Taenia solium C) Dipyllobotrium latum E) Echinococcus granulosus
B) Taenia saginata D) Dipylidium caninum

E 284. The adult form of this tapeworm has only four segments:
A) Taenia solium C) Dipyllobotrium latum E) Echinococcus granulosus
B) Taenia saginata D) Dipylidium caninum

C 285. The eggs of this worm are operculated:


A) Taenia solium B) Taenia saginata C) Dipyllobotrium latum D) Dipylidium caninum

B 286. Man is often infected by peeling water chestnuts with his teeth, which contain this organism:
A) Fasciola hepatica B) Fasciolopsis buski
C) Paragonimus weatermani D) Chlonorchis sinensis

A 287. This causes Egyptian hematuria:


A) Schistosoma hematobium C) Schistosoma japonicum
B) Schistosoma mansoni D) Fasciola hepatica

B 288. The eggs of this organism has a rudimentary lateral spine:


A) Schistosoma hematobium C) Schistosoma japonicum
B) Schistosoma mansoni D) Fasciola hepatica

D 289. This organism is acquired by eating infected raw fish:


A) Fasciola hepatica B) Fasciolopsis buski
C) Paragonimus weatermani D) Chlonorchis sinensis

C 290. This is also known as the Oriental ling fluke:


A) Fasciola hepatica B) Fasciolopsis buski
C) Paragonimus weatermani D) Chlonorchis sinensis

B 291. The presence of cryoglobulins in the serum of a patient may indicate that the patient has:
A) Anemia B) Circulating immune complexes
C) Hashimotos’s thyroiditis D) Pernicious anemia

B 292. The Coomb’s test is the most important laboratory method in the diagnosis of:
www.brex.us 294
MICROBIOLOGY & PUBLIC HEALTH

A) Myasthenia gravis C) Waldenstrom’s macroglubilemia


B) Autoimmune hemolytic anemia D) Rheumatoid arthritis

B 293. The most important antibody that plays a role in the pathogenesis of systemic lupus erythematosus is:
A) Antibody thyroglobuin C) Antibody to mitochondria
B) Antibody to DNA D) Antibody to smooth muscle

C 294. In the syndrome of post-streptococcal glomerulonephritis:


A) Streptococcal nucleases and streptolysin accumulate in the glomerular basement membrane.
B) Streptococcal capsular antigen (hyaluronic acid) and glucoronic acid subunits precipitate with
antibody and are deposited in the glomeruli in lumpy patterns.
C) Immunoglobulin and complement localize in the glomerular basement membrane.
D) Hematuria is due to the action of streptolysin O.

C 295. In group A beta hemolytic streptococci, the types are determined by the antigenic specificity of:
A) Capsule
B) Mucopeptide layer
C) M and/or T proteins
D) Extracellular products such as streptolysin O, which is produced only by group A streptococci

D 296. Congenital rubella can be diagnosed in a week-old infant by:


A) Demonstration of maternal IgM antibodies against rubella virus
B) Testing for HI antibodies specific for the virus in the infant’s serum
C) Demonstration of circulating IgG antibodies against rubella virus
D) Demonstration of rubella IgM antibodies in the infant

D 297. Caesarian section has been found to eliminate neonatal complications due to which of the following
viruses?
A) Varicella-zoster B) Cytomegalovirus C) Poliovirus D) Herpes simplex virus

A 298. Dermatophytes that infect special keratinized areas of the body, skin and nails only are likely to belong
to genus:
A) Epidermophyton B) Trichophyton C) Microsporum D) Trichosporum

B 299. A hapten is a substance that:


A) Induces cellular immune response but not antibody production
B) Does not induce any immune response when given alone but does elicit an immune response
when coupled to a larger molecule
C) Induces tolerance when given alone
D) When coupled to a larger molecule can be recognized by B lymphocytes but not T lymphocytes

B 300. Which of the following is a characteristic of the “positive strand” RNA virus?
A) The polymeranse contained in the virion is necessary in the replication.
B) The virion RNA can act as its own messenger RNA.
C) The virion RNA cannot be extracted in an infectious form.
D) Viral messenger RNA is complementary to the virion RNA.

www.brex.us 295
MICROBIOLOGY, PARASITOLOGY AND B. Xenograft
PUBLIC HEALTH (RED PACOP) C. Isograft
D. Autograft
1. The internationally recommended E. Transgraft
strategy for the control of tuberculosis
is DOTS, which stands for: 6. Which is true about major
A. Directly Observed Short-Term histocompatibility complex (MHC)?
Strategy A. Twins do not have the same
B. Directly Observed Treatment MHCs
Short Course B. Only identical twins have the same
C. Directly Observed Therapy Short MHCs
Course C. Tissues that produce self-antigens
D. Directly Observable Standard D. Not present in humans
Treatment E. Rejects T cell
E. Duration of Observed Treatment
Short Course 7. Viruses that infect E. Coli
A. Lytic
2. The five components of DOTS are the B. Lysogenic
following except: C. DNA phages
A. Sustained financial and political D. Coliphages
commitment E. Coliforms
B. Diagnosis by quality-ensured
sputum smear microscopy 8. Which of the ff. statements about
C. Standardized short course anti-TB Pneumocystis carinii is true?
drugs given to patients as A. Antibodies are found only in
supervised by treatment partners infected immunocompromised
D. Regular uninterrupted supply of hosts
quality anti-TB drugs B. It is a ricketssia
E. Supervision by DOH C. It can cause pulmonary
tuberculosis
3. Classification of diphtheria toxin D. Pathogenic for patients with AIDS
A. Neurotoxin with CD4 counts of > 800 cells/ml
B. Cytotoxin E. Diagnosis is through sputum
C. Enterotoxin smear microscopy
D. Endotoxin
E. Oxytoxin 9. Aspergillosis is caused by
A. Aspergillus flavus
4. The following are the characteristics of B. Aspergilloma dictus
inflammation except: C. Filobasidiella sp
A. Stomach ache D. Respiratory syncytial virus
B. Pain at the injured site E. Adenovirus
C. Redness
D. Increase in temperature 10. Which of the ff. diseases is a primary
E. Swelling zoonotic disease that is associated
with ingestion of contaminated milk?
5. A graft from twin brother to sister A. Anthrax
A. Allograft
B. Leptospirosis C. Western Blot Technique
C. Diphtheria D. Immunophoresis
D. Lyme disease E. Immunofluorescence
E. Brucellosis
16. Effective sterilization of a dental
11. A group of serum proteins that act in a instrument used in a patient reported
cascade manner resulting in microbial to be infected with AIDS, requires
cell lysis, inflammation and which of the ff. conditions:
opsonisation. A. Open flame
A. Complement system B. 100 °C for 10 mins.
B. Phagocytes C. 70% alcohol solution
C. Lymphatic system D. 3% H2O2 solution
D. Interferons
E. Natural killer cells 17. Ionizing radiation inactivates
vegetative bacteria by:
12. Antibody that is produced first in A. Driving away electrons & splitting
response to a primary infection the molecules into atoms or
A. IgG groups
B. IgA B. Inducing thymine dimmers
C. IgM C. Denaturing nucleoproteins
D. IgD D. Cross-linking sugar groups
E. IgE E. Inducing double-stranded breaks
in DNA

18. An effective membrane pore size


13. Antibody present in breastmilk necessary to filter out most rigid
A. IgG bacteria is approximately
B. IgA A. 25 µm
C. IgM B. 10 µm
D. IgD C. 1 µm
E. IgE D. 0.02 µm
E. 0.20 µm
14. Type of immunity resulting from
vaccination 19. A laboratory method used to show
A. Naturally acquired passive induced mutation
immunity A. Plate replication’
B. Naturally acquired active immunity B. Ames test
C. Artificially acquired passive C. Replica plating
immunity D. Velvet disk replication
D. Artificially acquired active E. Plaque assay
immunity
E. Adaptive immunity 20. Process of gene transfer responsible
for antibiotic resistance
15. A confirmatory test for HIV infection A. Transformation
A. ELISA B. Conjugation
B. Southern Blot Technique C. Transposon insertion
D. Frameshift mutation B. CUGGUGCUA
E. Both A & C C. CTGGUCGTA
D. CUGGACGUA
21. Algicide used in swimming pools E. CTGGUCGUA
A. Crystal violet
B. Gentian violet 26. Bacterial genes that may code for drug
C. Cupric sulfate resistance:
D. Methylene blue A. R factors
E. Ferric oxide B. F factors
C. Plasmid-encoded toxins
22. Lysol has a phenol coefficient of 5.0 D. Dissimilation plasmids
against Staphylococcus aureus. On E. Bacteriocins
the other hand, Formalin has a 0.5
phenol coefficient. Which of the 27. Which antibiotic acts on the 30S
statements below is incorrect? ribosomal subunit of bacteria and
A. Lysol is half as effective as interferes with the binding of tRNA to
Formalin ribosomes?
B. Lysol is 4.5 times more effective A. Chloramphenicol
than Formalin B. Streptomycin
C. Formalin is twice more effective C. Clindamycin
than phenol D. Erythromycin
D. Formalin is half as effective as E. Lincosamide
phenol
E. Formalin is 5x better than Lysol 28. Any mutant microorganism with a
nutritional requirement that is absent
23. A change in the DNA which results in in the parent microorganism is known
an insertion or deletion of one or more as
bases A. Mesotroph
A. Frameshift mutation B. Phototroph
B. Point mutation C. Auxotroph
C. Nonsense mutation D. Psychrotroph
D. Spontaneous mutation E. Halotroph
E. Induced mutation
29. The following is the bacterium used in
24. A stop codon that signals the end of the Ames test:
protein synthesis is A. Salmonella typhi
A. UUA B. Escherichia coli
B. UAG C. Bacillus subtilis
C. UGG D. Staphylococcus aureus
D. GAG E. Auxotrophic Salmonella
E. AUG typhimurium

25. What will be the transcribed sequence 30. Small segments of DNA that can move
for the ff. DNA sequence of a gene – from one region of a DNA molecule to
GACCTGCAT? another.
A. CTGGACGTU A. Plasmids
B. Genes D. 38 mols ATP/glucose mol
C. F factors E. 4 mols ATP/glucose mol
D. Transposons
E. R factors 36. Net yield of ATP molecules arising
from the complete oxidation of glucose
31. The first chemotherapeutic agent in fungal cells
effective against syphilis A. 2 mols ATP/ glucose mol
A. Penicillin B. 24 mols ATP/ glucose mol
B. Streptomycin C. 36 mols ATP/ glucose mol
C. Chloramphenicol D. 38 mols ATP/ glucose mol
D. Sulfonamide E. 4 mols ATP/ glucose mol
E. Salvarsan
37. A small segment of DNA that carries
32. The original source of penicillin genes for sex pili synthesis
discovered by Fleming was a mold A. R plasmids
known as B. F plasmids
A. Saccharomyces cerevisiae C. S plasmids
B. Penicillium notatum D. L plasmids
C. Penicillium chrysogenum E. RF plasmids
D. Penicillium flavus
E. Aspergillus flavus 38. Gene transfer between bacteria by
means of a virus
33. A biotechnological technique used to A. Transformation
treat disorders where a missing gene B. Conjugation
is inserted or a defective one is C. Transposon insertion
replaced in human cells D. Gene therapy
A. Gene therapy E. Transduction
B. Proteomics
C. Gene transfer 39. These are self-replicating circular
D. Genomics molecules of DNA carrying genes
E. Monoclonal antibody A. Plasmids
B. Genome
34. A process whereby an endospore C. Nucleotides
returns to its vegetative state is known D. Transposons
as E. Bacteriocins
A. Sporulation
B. Germination 40. A technique where a small samples of
C. Sporogenesis DNA can be quickly amplified for
D. Lysis analysis
E. Transformation A. Hybridoma
B. Recombinant DNA
35. Net yield of ATP molecules in aerobic C. Polymerase Chain Reaction
respiration of prokaryotes D. Cloning
A. 2 mols ATP/glucose mol E. Fluorescence
B. 24 mols ATP/glucose mol
C. 36 mols ATP/glucose mol
41. The relative amount of antibody in the 47. Recently discovered antimicrobial
serum is known as peptides produced by humans include
A. Antigen the following except:
B. Antibody titer A. Sideophores
C. Immunoglobulin level B. Dermicidins
D. Memory level C. Defensins
E. Antibody concentration D. Thrombocidins
E. Cathelicidins
42. The following is a macrolide antibiotic
A. Azithromycin 48. A group of serum proteins involved in
B. Streptomycin phagocytosis & lysis of bacteria
C. Kanamaycin A. Neutrophils
D. Gentamycin B. Opsonins
E. Neomycin C. Complement system
D. Immunoglobulins
43. The following is a streptogramin, a E. Major histocompatibility complex
unique group of antibiotics
A. Streptomycin 49. Fixed macrophages that protect and
B. Amikacin reside in the liver
C. Dalfopristin A. Osteoclast
D. Azithromycin B. Alveolar macrophages
E. Clarithromycin C. Kupffer cells
D. Histocytes
44. The Viridans streptococci group E. Sinusoids
includes the following except:
A. Streptococcus mutans 50. The quick response of the immune
B. Streptococcus mitis system due to recall by memory cells
C. Streptococcus pneumonia is known as
D. Streptococcus salivarius A. Anamnestic response
E. Streptococcus pyogenes B. Primary response
C. Memorable response
45. Cause of tuberculosis D. Immunologic response
A. Mycoplasma tuberculosis E. Humoral response
B. Mycobacterium tuberculosis ‘
C. Pseudomonas tuberculosis 51. Term for programmed cell death
D. Mycobacterium leprae A. Perforin
E. Mycoplasma pneumonia B. Apoptosis
C. Cytolysis
46. The enhancement of phagocytosis D. Inflammation
through coating with a complement E. Antigen presentation
A. Lysis
B. Complement activation 52. Referred to as inactivated toxins that
C. Cytolysis act as vaccines
D. Opsonization A. Exotoxins
E. Inflammation B. Endotoxins
C. Toxoids
D. Subunit vaccines
E. Inactivated vaccines 58. Discovered the microbial causes of
bacterial infections such as, anthrax
53. Duration of immunity of rabies vaccine and tuberculosis
is A. Robert Hooke
A. 2 years B. Anton Van Leeuwenhoek
B. 5 years C. Louis Pasteur
C. 10 years D. Robert Koch
D. 6 years E. Joseph Lister
E. 20 years
59. A sequence of experimental steps that
54. A monoclonal antibody used to treat help identify the specific microbe that
inflammatory conditions, such as, caused a specific disease.
rheumatoid arthritis and Crohn’s A. Germ Theory of Disease
disease B. Koch’s postulates
A. Trastuzumab C. Theory of Spontaneous
B. Rituximab Generation
C. Abciximab D. Theory of Immunity
D. Muromonab CD3 E. Transformation of Postulates
E. Infliximab
60. Coined the term “chemotherapy” and
55. A complement fixation diagnostic rest was among the first scientists who
for syphilis is searched for the “magic bullet”.
A. Wassermann test A. Paul Ehrlich
B. Widal test B. Alexander Fleming
C. Schick test C. Selman Waksman
D. Direct ELISA D. George Domagk
E. Indirect ELISA E. Heide Schulz

56. An amateur scientist who saw 61. The bacterium that can be used as an
“animalcules” in his single-lens agricultural pest control
microscope A. Escherichia coli
A. Robert Hooke B. Balantidium coli
B. Anton Van Leeuwenhoek C. Penicillin notatum
C. Louis Pasteur D. Aspergillus flavus
D. Robert Koch E. Bacillus thuringiensis
E. Joseph Lister
62. Collective term for microbes on and
57. His experiments on aseptic techniques inside human bodies
disapproved the Theory of A. Microbial count
Spontaneous Generation B. Too numerous to count
A. Robert Hooke C. Normal microbiota
B. Anton Van Leeuwenhoek D. Microbiofilm
C. Louis Pasteur E. Biological flora
D. Robert Koch
E. Joseph Lister
63. Term for new or changing diseases
that are increasing or have the 68. A method used to identify bacteria in a
potential to increase in incidence in sample without culturing the bacteria
the near future A. Flow cytometry
A. Neglected Diseases B. Phage typing
B. Emerging infectious diseases C. Immunofluorescence
C. Non-emerging infectious diseases D. Fluorescent in situ Hybridization
D. Rare diseases E. ELISA
E. Degenerative diseases
69. A reference for taxonomic
classification scheme for prokaryotes
64. The cause of avian influenza A is A. Laboratory Guide in Microbiology
A. H5N1 virus B. Pharmaceutical Microbiology
B. H1N1 virus C. Botanical taxonomy
C. H1N3 virus D. Bergey’s Manual of Systematic
D. H3N1 virus Bacteriology
E. HIV E. Bergey’s Manual of Determinative
Bacteriology
65. Which of the following is n example of
bioremediation? 70. A reference for the identification of
A. Application of oil-degrading bacteria based on criteria, such as,
bacteria to an oil spill morphology, cell wall composition,
B. Application of bacteria to a crop to oxygen requirements etc.
prevent frost damage A. Laboratory Guide in Microbiology
C. Fixation of gaseous nitrogen into B. Pharmaceutical Microbiology
usable nitrogen C. Botanical Taxonomy
D. Production by bacteria of a human D. Bergey’s Manual of Systematic
protein like interferon Bacteriology
E. All of the above. E. Bergey’s Manual of Determinative
Bacteriology
66. The movement of eukaryotic
cytoplasm from one part of the cell to 71. Founder of the five-kingdom system
another which helps distribute where the prokaryotes were placed in
nutrients is Kingdom Prokaryotae and eucaryotes
A. Brownian movement compromised the other four kingdoms,
B. True motility A. Carolus Linnaeus
C. Pseudo motility B. Robert Murray
D. Cellular motility C. Carl Woese
E. Cytoplasmic streaming D. Robert Whittaker
E. Charles Darwin
67. Cause of Lyme disease
A. Borrelia burgdorferi 72. The following are true about viruses
B. Candida albicans except
C. Salmonella typhi A. Has a single type of nucleic acid,
D. Limulus polyphemus either DNA or RNA
E. Treponema pallidum B. Multiply only inside living host cells
C. Considered obligate intracellular 78. Type 1 hypersensitivity reactions that
parasites occur immediately upon re-exposure
D. Contains a protein coat that of a sensitized individual to an antigen
surrounds the nucleic acid A. Delayed hypersensitivity reactions
E. Can be infected by bacteria B. Cytotoxic reactions
C. Anaphylactic reactions
73. Protein coat that protects the nucleic D. Immune complex reactions
acid of a virus E. Cell-mediated reactions
A. Plasmid
B. Capsid 79. Hemolytic disease of the newborn will
C. Capsomere occur in this condition:
D. Envelope A. Second or higher pregnancy of
E. Spike sensitized Rh(-) Mother, Rh (+)
baby with an Rh(+) father
74. The following are steps in the B. Second or higher pregnancy of
occurrence of inflammation except: Rh(+) mother, Rh(+) baby with an
A. Tissue response Rh(-) father
B. Production of interferon C. Second or higher pregnancy of
C. Phagocyte migration sensitized Rh(+) Mother, Rh (+)
D. Tissue repair baby with an Rh(-) father
E. Vasodilation D. Second or higher pregnancy of
Rh(-) mother, Rh(+) baby with an
75. The following are involved in the Rh(+) father
increase in abnormally high body E. Second or higher pregnancy of
temperature or fever except: Rh(-) mother, Rh(+) baby with an
A. Endotoxins Rh(-) father
B. Exotoxins
C. Lipopolysaccharides 80. The following are true about
D. Interleukin-1 immunosuppressants except:
E. TNF-α A. Promotes immune response to
protect the transplant
76. The following are RNA viruses except B. Suppress normal immune
A. HIV response against transplant
B. Flu virus C. Include monoclonal antibodies
C. Rabies D. Include cyclosporine
D. Chicken pox virus E. Blocks secretion of interleukin 2 to
E. Ebola virus disrupt cell-mediated immune
response.
77. The following are DNA viruses except
A. HAV 81. Human immunodeficiency virus
B. Shingles virus targets which cell in the immune
C. HBV system?
D. Herpes simplex type 2 virus A. CD4+ T helper cell
E. Herpes simplex type 1 virus B. B cell
C. CD8+B helper cell
D. CD4+ T cytotoxic cell’
E. Natural killer cell
87. A biological vector
82. A condition where antibodies cause A. Anopheles mosquito
antigens to clump together . B. Cockroach
A. Opsonisation C. Housefly
B. Phagocytosis D. Aedes aegypti
C. Complement fixation E. Termite
D. Neutralization
E. Agglutination 88. In the host-parasite relationship, the
mosquito vector of Plasmodium sp
83. Immune response that defends the acts as the
body against fungi, parasites, cancer A. Definitive host
and transplants. B. Intermediate host
A. Cell-mediated C. Parasitic host
B. Humoral D. Insect host
C. Antibody-mediated E. Pathogen
D. B cell-mediated
E. Memory B-cell mediated 89. The following statements about
resistance to antimicrobial drugs are
84. A nosocomial pathogen that normally true except:
inhabits the skin and commonly A. Resistance to chloramphenicol is
contaminates prosthetic devices such due to enzyme that acetylates the
as, catheters and heart valves, leading drug
to symptomatic microbemia B. Resistance to penicillin is due to
A. Staphylococcus aureus reduced affinity of transpeptidases
B. Staphylococcus epidermis C. Resistance to penicillin is known
C. Streptococcus aureus to be due to cleavage of β-
D. Streptococcus pyogenes lactamase
E. Streptococcus mutans D. Resistance to tetracycline is due
to an enzyme that hydrolyzes the
85. What is included in the indicator ester linkages
system of a complement fixation test? E. Resistance to anti-TB drugs is due
A. Specific antibody & complement to mutation of the bacteria
B. Red blood cells & hemolysin
C. Heat-inactivated patient’s serum 90. The mechanisms of antibiotic
D. Guinea pig serum resistance include the following
E. Rat red blood cells except:
A. Preventing the entry of the drug to
86. In malaria, the form of plasmodia that the target site within the microbial
is transmitted from mosquito to human cell
is the B. Alteration of the drug’s target site
A. Gametocyte C. Rapid entry of the antibiotic to the
B. Merozoite cell
C. Oocyst D. Inactivation of the drug by
D. Trophozoite microbial enzymes
E. Sporozoite
E. Mutation of the bacteria due to 96. The interval between initial infection
antibiotic misuse and the first appearance of any signs
or symptoms is known as
91. This pathogen is a commen cause of A. Prodromal period
nosocomial infections B. Infection period
A. Pseudomonas aeruginosa C. Incubation period
B. Bacillus subtilis D. Period if illness
C. Salmonella cerevisiae E. Period of convalescence
D. Clostridium tetani
E. Rabies virus 97. Time period that the person regains
strength from infection and the body
92. According to the Center for Disease returns to its pre-diseased state
Control and Prevention, the single A. Prodromal period
most important means of preventing B. Infection period
the spread of nosocomial infection is C. Incubation period
A. Education on microbial control D. Period of illness
B. Disinfection of hospital E. Period of convalescence
instruments and devices
C. Use of hand gel 98. A relatively short period of occurrence
D. Handwashing of disease characterized by early mild
E. Placing face masks on al infected symptoms of the disease, such as,
patients body ache, headache and malaise.
A. Prodromal period
93. The following are emerging infectious B. Infection period
diseases starting year 2000 except: C. Incubation period
A. H1N1 influenza D. Period of illness
B. HIV-AIDS E. Period of convalescence’
C. Anthrax
D. Chagas disease 99. Rabies virus is transmitted to humans
E. Whooping cough through
A. Direct contact transmission
94. Emerging infectious diseases can be B. Indirect contact transmission
due to C. Droplet transmission
A. Climatic changes D. Vehicle transmission
B. Too many vaccines E. Mechanical ventilator transmission
C. Inefficient case reporting
D. Rational antibiotic use 100. Congenital infection, like rubella,
E. Surveillance of DOH occurs as a result of
A. Viral toxins crossing the placenta
95. A continuous source of the diseases B. Viral toxins impairing maternal
organism is known as the nourishment of the fetus
A. Carrier C. Transplacental spread of virus and
B. Reservoir of infection growth in the fetus
C. Deep wells D. Autoimmune response of maternal
D. Hospital cells against fetal tissue
E. Secondary use E. Child naturally has the infection
C. Lipid O
101. An enzyme produced by virulent D. Lipid A
bacteria to promote the spread of the E. Oligosaccharide
bacteria to the host and otherwise
known as the spreading factor 106. A life-threatening decrease in blood
A. Collagenase pressure caused by bacteria is
B. Streptokinase referred to as
C. Protease A. Shock
D. Hyaluronidase B. Technical knock-out
E. Hemolysin C. Septic shock
D. Hypotension
102. Hemolysins can lyse red blood cells. E. Anemia
Which of the following is not correct?
A. α- hemolysin produces greenish 107. An endotoxin producing bacterium
ring around bacterial colonies on A. Neisseria meningitidis
blood agar plate B. Escherichia coli
B. β- hemolysin produces clear ring C. Clostridium perfringens
around colonies on blood agar D. Vibrio cholera
plate E. Clostridium tetani
C. γ- hemolysin produces no visible
change around colonies on blood 108. Inclusion bodies produced by rabies
agar plate virus
D. Streptococcus pyogenes has β- A. Syncytial bodies
hemolysin B. Negri bodies
E. Staphylococcus aureus has γ- C. Black bodies
hemolysin D. Interferons
E. Cytopathic areas
103. Exotoxins can lead to the following
diseases except 109. Term for the characteristic of the
A. Rabies disease that can be observed by
B. Tetanus examining the patient
C. Diphtheria A. Symptom
D. Botulism B. Syndrome
E. Cholera C. Observation
D. Sequelae
104. Botulinum toxin is classified as a E. Sign
A. Cytotoxin
B. Enterotoxin 110. Term for the characteristic of the
C. Neurotoxin disease that can be observed or felt
D. Endotoxin only by the patient
E. Phytotoxin A. Symptom
B. Syndrome
105. The endotoxin or the lipid portion of C. Observation
the lipopolysaccharide is called the D. Sequelae
A. Lipid C E. Sign
B. Lipid B
111. Term for the combination of signs A. Raynaud’s phenomenon
and symptoms that occur together and B. Q fever
indicate a particular disease C. Reye’s syndrome
A. Symptom D. Rocky mountain spotted fever
B. Syndrome E. Gray baby syndrome
C. Observation
D. Sequelae 117. This is the only Bacillus species that
E. Sign is highly pathogenic for animals and
humans and is the causative agent for
112. An example of an acute infectious anthrax
disease A. B stearothermophilus
A. Common cold B. B cereus
B. Leprosy C. B thuringiensis
C. Tuberculosis D. B subtilis
D. Diphtheria E. B anthracis
E. Gingivitis
118. Lesions of the oral cavity which
113. According to severity, TB may be manifest as tiny red patches with
classified as central white specks in patients
A. Subacute and latent infected with Rubeola virus
B. Chronic and latent A. Erythema infectiosum
C. Sporadic and latent B. Macular rash
D. Chronic C. Fever blisters
E. Latent D. Koplik’s spots
E. Negri bodies
114. A secondary infection that results
from destruction of normal microflora 119. During the first 10 minutes or so after
and often follows the use of broad- injection of phage DNA, no phage can
spectrum antibiotics be recovered by disrupting the
A. Mixed infection infected bacterium. This is termed as:
B. Subclinical infection A. Eclipse period
C. Inapparent infection B. Latent period
D. Superinfection C. Rise period
E. Co-morbid infection D. Burst size
115. These are generally benign skin E. Replication cycle
growths caused by viruses
A. Bullae 120. All are major causes of bacterial
B. Warts meningitis except:
C. Boils A. Haemophilus influenzae
D. Acne B. Neisseria meningitidis
E. Papules C. Hib
D. Listeria monocytogenes
116. A severe complication of chickenpox, E. Streptococcus pneumoniae
influenza, and other viral diseases
which affects children and is 121. This gram negative bacteria
aggravated by the use of aspirin produces a water-soluble blue
pigment, pyocyanin, and a water- cultivated plants. These are also the
soluble fluorescent pigment, pyoverdin smallest known agents of infectious
A. Pseudomonas aeruginosa disease.
B. Pseudomonas mallei A. Virion
C. Pseudomonas pseudomallei B. Bacteriophages
D. Pseudomonas syringae C. Paschen bodies
E. Pseudomonas fluorescens D. Viroids
E. Oncogenes
122. These bacteria differ from most other
enterobacteriaceae by being mainly 127. The most common causative agent
associated with plants of genital herpes
A. Proteus A. HSV-1
B. Erwinia B. HSV-2
C. Shigella C. HSV-3
D. Serratia D. HSV-4
E. Yersinia E. HSV-6

123. Which of the following viruses is a 128. Theses are common bread molds
DNA virus? which cause much food spoilage and
A. Bunyaviridae grow on bread, vegetables, fruits and
B. Adenoviridae other food products.
C. Reoviridae A. Mucor
D. Togaviridae B. Saccharomyces
E. Orthomyxoviridae C. Rhizopus
D. Schizosaccharomyces
124. The following RNA viruses are helical E. Agaricus
in shape, except:
A. Rhabdoviridae 129. Streptomyces griseus is the source
B. Arenaviridae of this antibiotic.
C. Bunyaviridae A. Griseofulvin
D. Orthomyxoviridae B. Streptomycin
E. Picornaviridae C. Erythromycin
D. Kanamycin
125. A patient with nausea, vomiting, and E. Chlorampenicol
diarrhea within 5 hours after eating
most likely has: 130. These antimicrobial drugs differ from
A. Shigellosis the antibiotics in that they do not occur
B. Cholera naturally. The prototype of these is
C. E. coli gastroenteritis furfural, an aldehyde derivative.
D. Salmonellosis A. Interferons
E. Staphylococcus food poisoning B. Tetracyclines
C. Sulfonamides
126. These are nucleic acid entities of D. Nitrofurans
relatively low molecular weight (1.1 to E. Anthramycin
1.3 x 105) and unique structure that
cause several important diseases of
131. The following organisms are good greatly reduced by the use of this
recipients of R factor from E. coli drug.
donor, except: A. Sibromycin
A. Enterobacter B. Anthramycin
B. Proteus C. Sulfaguanidine
C. Klebsiella D. Acycloguanosine
D. Salmonella E. Amantadine
E. Shigella
136. This drug is member of the allylamine
132. This is the first known disease for class of antimycotics and inhibits the
which a chemotherapeutic agent was enzyme squalene epoxidase in fungal
used. ergosterol biosynthesis.
A. Mucormycosis A. Clotrimazole
B. Bubonic plague B. Terbinafine
C. Syphilis C. Linezolid
D. HSV-1 D. Flucytosine
E. Gonorrhea E. Nalidixic acid

133. It inhibits peptidoglycan synthesis by 137. This is the commonest cause of


binding D-alanyl-D-alanine group on pneumonia and still responds well to
the peptide side chain of one of the penicillin despitea global in isolates
membrane-bound intermediates. showing reduced susceptibility to this
A. Vancomycin agent.
B. Bacitracin A. Legionella pneumophila
C. Methicillin B. Mycoplasma pneumonia
D. Chlortetracycline C. Streptococcus pneumoniae
E. Cycloserine D. Haemophilus influenza
E. Streptococcus pyogenes
134. Because it is a structural analog of
pyridoxine, or Vitamin B6 and 138. The following are fungal infections of
nicotinamide, it can block pyridoxine the skin and nails, except:
and nicotinamide-catalyzed reactions A. Tinea pedis
which may account for its antimicrobial B. Tinea capitis
activity. C. Cryptococcus neoformans
A. Isonicotinic Acid Hydrazide D. Tinea corporis
B. Nalidixic acid E. Malassezia furfur
C. Isoniazid 139. This drug is thought to block the
D. A and B assembly of arabinogalactan
E. A and C polysaccharide by inhibition of an
arabinotranferase enzyme.
135. This is a low-molecular weight A. Rifampin
compound which is very effective B. Isoniazid
against influenza A virus but not C. Pyrazinamide
against influenza B virus. The D. Ethambutol
incidence of influenza A infections is E. Streptomycin
140. These are structural analogues of E. Enteropathogenic
PABA that competitively inhibit the
incorporation of PABA into
dihydropteroic acid. 145. Which of the following is not a
A. Sulfonamides tickborne disease?
B. Nalidixic acid A. Babesiosis
C. Cephalosporins B. Ehrlichiosis
D. B and C C. Lyme disease
E. A and C D. Relapsing fever
E. Tularemia
141. Which of the following is not
matched? 146. Causative agent for Undulant fever.
A. Antihelminthic - inhibition of cell A. Bartonella henselae
wall synthesis B. Borrelia burgdorferi
B. Antibacterial – inhibition of nucleic C. Brucella melitensis
acid synthesis D. Bacillus anthracis
C. Antiviral – inhibition of DNA E. Bordetella pertussis
synthesis
D. Antifungal – inhibition of mitosis 147. This refers to the uncontrolled
E. Antifungal – injury to plasma proliferation of microbes in the blood.
membrane A. Sepsis
B. Septicemia
142. Characteristics of the causative C. Septic shock
agent of tetanus, except: D. Anaphylaxis
A. Gram positive E. Bacteremia
B. Produces endospores
C. Obligate aerobe 148. This refers to inflammation of the
D. Transmitted through contaminated urinary bladder.
soil A. Cystitis
E. Produces a potent neurotoxin B. Urethritis
C. Ureteritis
143. Viral disease characterized by D. Pyelonephritis
inflammation of the parotid glands, E. Nephritis
fever, and pain during swallowing.
A. Tonsilitis 149. Disease-causing microorganisms are
B. Gingivitis called:
C. Measles A. Vectors
D. Mumps B. Bacteria
E. Typhoid fever C. Fomites
D. Pathogens
144. Strain of E. coli that can produce a E. Toxins
Shigella-like dysentery.
A. Enterotoxigenic 150. Vector-borne diseases,except:
B. Enteroinvasive A. Trypanosomiasis
C. Enterohemmorhagic B. Lyme disease
D. Enterohemolytic C. Cryptosporidiosis
D. Yellow fever C. Sulfur
E. Plague D. Phosphorus
E. None of the above
151. This type of microscopy does not
require fixing or staining of the 156. Which of the following enzymes
specimen, and also gives a detailed neutralize/s various toxic forms of
examination of internal structures in oxygen?
living microorganisms by using slight A. Superoxide dismuthase
variations in refractive index. B. Catalase
A. Phase-contrast microscopy C. Peroxidase
B. Compound light microscopy D. A, B and C
C. Darkfield microscopy E. None of the above
D. Fluorescence microscopy
E. Both A and C 157. Which of the following is an obligate
anaerobe?
152. What is/are the staining method/s A. Micrococcus luteus
used to identify all bacteria in the B. Staphylococcus aureus
genus Mycobacterium and the disease C. Escherichia coli
producing strains of Nocardia. D. Clostridium perfringens
A. Simple staining E. Mycobacterium tuberculosis
B. Gram staining
C. Acid-fast bacilli 158. A type of medium with known
D. Negative staining chemical composition used for
E. Both B and C microbiological assays.
A. Selective media
153. Which of the following dyes cannot B. Differential media
be used in negative staining? C. Complex media
A. Eosin D. Anaerobic Media
B. Nigrosin E. Chemically defined media
C. India ink
D. Safranin 159. This type of media suppresses
E. Both A and D growth of unwanted microbes and
encourage growth of desired
154. These are organisms responsible for microbes.
low temperature food spoilage. A. Enrichment media
A. Psychrotrophs B. Differential media
B. Psychrophiles C. Selective media
C. Thermophiles D. Reducing media
D. Mesophiles E. Chemically defined media
E. Both A and C
160. Which of the following is an indirect
155. Which of the following elements method of estimating bacterial
is/are NOT needed by microbes for number?
the synthesis of cellular materials? A. Turbidity method
A. Nitrogen B. Serial dilution
B. Carbon C. Most probable number
D. Filtration 166. Which of the following is used as a
E. Direct microscopic count gaseous chemosterilizer?
A. Phenol
161. Process of destroying vegetative B. Ethylene oxide
pathogens but not necessarily C. Ethanol
endospores or viruses, usually or D. Ammonia
inanimate objects. E. Benzalkonium chloride
A. Sterilization
B. Antisepsis 167. Which of the following is not a
C. Sanitization spirochete?
D. Degerming A. Treponema
E. Disinfection B. Leptospira
C. Vibrio
162. Removal of transient microbes from D. Borrelia
the skin by mechanical cleansing or by E. None of the above
the use of an antiseptic.
A. Sterilization 168. Which of the following bacteria are
B. Antisepsis spore formers?
C. Sanitization A. Bacillus
D. Degerming B. Clostridium
E. Disinfection C. Salmonella
D. Both A and B
163. Which of the following is/are an E. Both A and C
effective method of sterilization?
A. Refrigeration 169. Which of the following is not a gram
B. Pasteurization negative facultatively anaerobic rod-
C. Direct flaming shaped bacteria?
D. Autoclaving A. Escherichia
E. Both C and D B. Salmonella
C. Shigella
164. Chemical used for microbial control D. Enterobacter
by Joseph Lister. E. NOTA
A. Iodine
B. Phenol 170. Which of the following is not a
C. Carbolic acid characteristic of the genus Neisseria?
D. Chlorinated Lime A. Diplococcic
E. Both B and C B. Obligate anaerobe
C. Some members cause meningitis
165. Which of the following metals has the D. Some members causes gonorrhea
strongest oligodynamic action? E. NOTA
A. Silver
B. Mercury 171. Which of the following is transmitted
C. Iron via the fecal-oral route?
D. Copper A. HAV
E. Zinc B. HBV
C. HCV
D. HDV D. Lyophilization involves desiccation
E. Both A and B in high temperature
E. NOTA
172. Which of the following is not a
characteristic of Fungi? 177. Which of the following is the
A. Chemoheterotroph preferred method of sterilizing liquids
B. Unicellular containing toxins, vaccines and
C. Multicellular enzymes?
D. Photoautotroph A. Pasteurization
E. Both B and D B. Autoclaving
C. Filtration
173. Which of the following types of D. Refrigeration
asexual spore is formed within a sac E. NOTA
at the end of an aerial hypha?
A. Blastospore 178. Which type of media is used to grow
B. Chlamydospore obligate anaerobes?
C. Endospore A. Complex media
D. Conidiospore B. Enrichment media
E. Arthrospore C. Differential media
D. Selective media
174. Which of the following is typically E. Reducing media
observed as microscopic grapelike
clusters and yellow-pigmented 179. Aerobes contain this enzyme which
colonies? converts superoxide to oxygen and
A. Staphylococcus aureus hydrogen peroxide.
B. Streptococcus pneumonia A. Catalase
C. Clostridium botulinum B. Superoxide dismutase
D. Neisseria gonorrhea C. Peroxidase
E. Both A and B D. Both B and C
E. NOTA
175. Which of the following is/are true
about moist heat sterilization? 180. This group of anaerobes can grow in
A. An autoclave can be used the presence of oxygen and are not
B. Kills vegetative cells, spores and usually harmed by its presence in the
viruses environment.
C. Works by coagulation of proteins A. Strict anaerobes
D. A and C only B. Obligate anaerobes
E. A, B and C C. Aerotolerant anaerobes
D. Facultative aerobes
176. Which of the following is true? E. Both A and B
A. Pasteurization is a form of
sterilization 181. These microorganisms have an
B. Dry heat destroys cells by optimal growth between -5°C and
oxidation 20°C and which can be found in the
C. Filtration cannot be used for supercooled waters of the arctic and
sterilization Antarctic.
A. Psychrophiles 186. In Reductase Test using milk, the
B. Mesophiles time it takes for methylene blue to
C. Thermophiles become colorless is the:
D. Hyperthermophiles A. Methylene blue oxidation time
E. NOTA B. Methylene blue reaction time
C. Methylene blue catalysis time
182. This is the time required to kill a D. Methylene blue reduction time
suspension of cells or spores at a E. Methylene blue disappearance
given temperature. time
A. Thermal death point
B. Thermal death time 187. These are indicator bacteria which
C. Burst size are found in the intestines of humans
D. Incubation period and warm-blooded animals, ferment
E. Triple death point lactose to produce acid and gas, and
its presence in water suggests the
183. This is also known as Mixed-Acid potential for disease. Examples of
Fermentation. such organisms are E. coli and
A. Voges-Proskauer test Enterobacter aerogenes.
B. Citrate test A. Enterics
C. Methyl red test B. Coliforms
D. 2,3 – Butanediol Fermentation test C. Colony forming units
E. Both A and D D. Enterococcus
E. NOTA
184. The bacteria that are mixed-acid
fermenters generally produce gas 188. Also known as Ziehl-Neelsen
because they elaborate this enzyme Method:
which splits formic acid to produce A. Gram staining
CO2 and H2O. B. Simple staining
A. Formic hydrogenylase C. Acid-fast staining
B. Nitrate reductase D. Negative staining
C. Formicase E. Capsular staining
D. Catalase
E. NOTA 189. Which of the following organisms can
be identified using the answer in
185. This is a multiple test medium that number 188?
will detect the fermentation of glucose A. Mycoplasma pneumoniae
and lactose and the production of B. Mycobacterium leprae
hydrogen sulphide resulting from the C. Mycobacterium tuberculosis
breakdown of the amino acid cysteine. D. Both A and C
A. SIM medium E. Both B and C
B. Litmus Milk
C. Hydrogen sulfide 190. Property of an ideal antimicrobial
D. Kligler’s Iron Agar agent
E. NOTA A. Production of drug resistance
B. Selective toxicity
C. Toxic to the host cell
D. Broad spectrum of activity B. Amphotericin B
E. Produces hypersensitivity C. Terbinafine
D. Fluconazole
191. Drugs that inhibit the 50s portion of E. Caspofungin
the bacterial ribosome, except:
A. Streptomycin 197. Which of the following drugs is most
B. Erythromycin resistant to the action of penicillinase?
C. Chloramphenicol A. Methicillin
D. Quinupristin B. Amoxicillin
E. Clindamycin C. Benzathine penicillin
D. Ticarcillin
192. Microbial source of tetracycline: E. Mezlocillin
A. Streptomyces erythreus
B. Streptomyces griseus 198. The role of cilastatin sodium when
C. Streptomyces aureofaciens combined with imipenem is:
D. Streptomyces nodosus A. It protects against the action of
E. Streptomyces roseosporus beta-lactamase
B. It prevents degradation of
193. Co-trimoxazole is a combination of: imipenem in the kidneys
A. Sulfamethoxazole and clavulanic C. It exerts antimicrobial action to
acid broaden the spectrum of activity
B. Sulfathiazole and trimethoprim D. It potentiates the antimicrobial
C. Sulfamethoxazole and action imipenem
trimethoprim E. It facilitates the absorption of
D. Sulfathiazole and clavulanic acid imipenem
E. None of the above
199. The following aminoglycosides are
194. Fluoroquinolones exert their derived from Streptomyces, except:
antimicrobial action by inhibiting: A. Streptomycin
A. Topoisomerase I B. Tobramycin
B. Topoisomerase II C. Gentamicin
C. Topoisomerase IV D. Neomycin
D. A &B E. Spectinomycin
E. B &C
200. This specie of Plasmodium causes
195. Trimethoprim is an inhibitor of which the most dangerous form of malaria.
enzyme: A. P. falciparum
A. Dihydropteroate synthase B. P. ovale
B. Dihydrofolate synthase C. P. vivax
C. Dihydropteroate reductase D. P. malariae
D. Dihydrofolate reductase E. None of the above
E. None of the above
201. Refers to a continuum of learning
196. A synthetic antifungal agent which which enables people, as individuals
inhibits squalene epoxidase: and as members of social structures,
A. Griseofulvin to voluntarily make decisions, modify
behaviors and change social of behavior change to another as their
conditions in ways which are health cifidence in their ability to make this
financing. change increases.
A. Health promotion A. Health belief model
B. Health education B. Social cognitive theory
C. Health perception C. Transtheoretical model
D. Health behavior D. Theory of reasoned action
E. Health beliefs E. Ecologic model

202. Refers to the aggregate of all 206. In the Health Belief Model, these are
purposeful activities designed to the messages or triggers that will
improve personal and public health activate the individual to take action.
through a combination of strategies. A. Perceived susceptibility
A. Health promotion B. Cues action
B. Health education C. Perceived barriers
C. Health perception D. Self-efficacy
D. Health behavior E. Contemplation
E. Health beliefs
207. In the Health Belief Model, this refers
203. The health behavior model that to individual’s level of confidence that
explains that health behavior affects he/she can successfully carry out a
and is affected by multiple levels of particular behavior.
influence of individual, interpersonal, A. Perceived susceptibility
institutional, community and public B. Cues action
policy factors, and that health behavior C. Perceived barriers
shapes and is shaped by the social D. Self-efficacy
environment. E. Contemplation
A. Health belief model
B. Social cognitive theory 208. A health behavior model that
C. Transtheoretical model suggests that people make changes in
D. Theory of reasoned action their behaviors on the basis of their
E. Ecologic model belief that the intended outcome of a
behavior change is positive or
204. A health behavior model that negative.
examines perceptions of the benefits A. Health belief model
of avoiding the health threat and B. Social cognitive theory
factors influencing the decision to act C. Transtheoretical model
on it. D. Theory of reasoned action
A. Health belief model E. Ecologic model
B. Social cognitive theory
C. Transtheoretical model 209. In theory of planned behavior, this
D. Theory of reasoned action refers to individual’s evaluation of a
E. Ecologic model behavior.
A. Subjective norm
205. A health behavior model that tackles B. Behavioral control
individual’s movement from one stage C. Attitude
D. Action B. Strengthen community actions
E. Contemplation C. Develop personal skills
D. Build healthy public policy
210. In theory of planned behavior, this E. Reorient health services
refers to the individual’s perception
that he/she is likely to perform the 214. In the Ottawa Charter, it is an action
behavior. area which explains that health
A. Public policy promotion should support personal
B. Behavioral intention and social development through
C. Precontemplation providing information, education for
D. Perceived benefits health, and enhancing life skills.
E. Preparation A. Create supportive environments
B. Strengthen community actions
211. In the Ottawa Charter, it is an action C. Develop personal skills
area for health promotion that D. Build healthy public policy
combines diverse but complementary E. Reorient health services
approaches including legislation, fiscal
measures, taxation, and 215. In the Ottawa charter, it is an action
organizational change. area that must lead to a change of
A. Create supportive environments attitude and organization of health
B. Strengthen community actions services which refocuses on the total
C. Develop personal skills needs of the individual as a whole
D. Build healthy public policy person.
E. Reorient health services A. Create supportive environment
B. Strengthen community actions
212. In the Ottawa Charter, it is an action C. Develop personal skills
area which explains that health D. Build healthy public policy
promotion should generate living and E. Reorient health services
working conditions that are safe,
stimulating, satisfying and enjoyable. 216. Defined as “ what we as a society do
A. Create supportive environments to assure the conditions in which
B. Strengthen community actions people can be healthy”.
C. Develop personal skills A. Sustainability
D. Build healthy public policy B. Social marketing
E. Reorient health services C. Intersectoral participation
D. Public health
213. In the Ottawa Charter, it is an action E. Community involvement
area which explains that community
development draws an existing human 217. Which of the following is NOT a
and material resources in the macro-level public health activity?
community to enhance self-help and A. Community diagnosis
social support, and to develop flexible B. Patient-directed services
systems for strengthening public C. Policy development
participation in and direction of health D. Health planning
matters. E. Partnership establishment
A. Create supportive environments
218. A spectrum of diseases arising from A. Goiter
a diet that is poor in protein and B. Anemia
calories, especially in children under C. Cretinism
five. D. Parasitism
A. Iodine deficiency disorders E. Xerophthalmia
B. Marasmus
C. Protein energy malnutrition 223. A common preventable lifelong
D. Kwashiorkor mental and physical retardation in the
E. Xerophthalmia fetus and infant brought about by
iodine deficiency.
219. This stems from an inadequate A. Goiter
calorie intake due to insufficient diet, B. Anemia
improper feeding habits, metabolic C. Cretinism
abnormalities or congenital D. Parasitism
malformations that result to gain E. Xerophthalmia
weight, followed by loss of weight until
emaciation occurs. 224. Causative agent for genital herpes,
A. Iodine deficiency disorders or the anogenital vesicular lesions and
B. Marasmus ulcerations
C. Protein energy malnutrition A. Human papilloma virus
D. Kwashiorkor B. Molluscum contagiosum virus
E. Xerophthalmia C. Phthirus pubis
D. Herpes simplex virus
220. Malnutrition type which is due to E. Trichomonas vaginalis
deprivation of sufficient quality protein
foods, impaired absorption of protein, 225. Causative agent for the public lice
or abnormal losses of protein from the infestation, which are commonly called
body, which also later results to fatty “crabs”.
liver. A. Human papilloma virus
A. Iodine deficiency disorders B. Molluscum contagiosum virus
B. Marasmus C. Phthirus pubis
C. Protein energy malnutrition D. Herpes simplex virus
D. Kwashiorkor E. Trichomonas vaginalis
E. Xerophthalmia
226. Causative agent genital warts.
221. The term that describes all the eye A. Human papilloma virus
signs of Vitamin A deficiency if it B. Molluscum contagiosum virus
reaches a critically low level. C. Phthirus pubis
A. Iodine deficiency disorders D. Herpes simplex virus
B. Marasmus E. Trichomonas vaginalis
C. Protein energy malnutrition ‘
D. Kwashiorkor 227. Causative agent of chancroid, or the
E. Xerophthalmia painful genital ulcers which are
accompanied by inguinal swelling or
222. Thyroid enlargement due to iodine “bubo”.
deficiency. A. Molluscum contagiosum virus
B. Chlamydia trachomatis D. Administration of parenteral
C. Haemophilus ducreyi oxytocic drugs
D. Trichomonas vaginalis E. Provision of surgical delivery
E. Klebsiella granulomatis (caesarian section)

228. Causative agent of Granuloma 232. Which of the following is not an


inguinale, or Donovanosis. emergency neonatal care given in a
A. Molluscum contagiosum virus Comprehensive Emergency Obstetric
B. Chlamydia trachomatis and Newborn Care (CEMONC)
C. Haemophilus ducreyi facility?
D. Trichomonas vaginalis A. Newborn resuscitation
E. Klebsiella granulomatis B. Oxygen support
C. Antenatal administration of
229. Refers to the death of a woman while (maternal) steroids for threatened
pregnant or within 42 days of premature delivery
termination of pregnancy, irrespective D. Treatment of neonatal sepsis
of the duration or site of the infection
pregnancy, from any cause related to E. None of the above
or aggravated by the pregnancy or its
management, but not from accidental 233. What is the Millennium Development
causes. Goal #5?
A. Morbidity rate A. Promote gender equality
B. Infant mortality B. Improve maternal heath
C. Maternal death C. Achieve universal primary
D. Mortality rate education
E. Population pyramid D. Eradicate extreme poverty
E. Reduce child mortality
230. Which is not part of the Full Package
in Reproductive Health? 234. What is the Millennium Development
A. Family planning Goal #4?
B. Diagnosis and treatment of breast A. Promote gender equality
cancer B. Improve maternal health
C. Antenatal care C. Achieve universal primary
D. Support for exclusive education
breastfeeding D. Eradicate extreme poverty
E. None of the above E. Reduce child mortality

231. Which of the following is not part of 235. Refers to both macronutrient
the services of the Basic Emergency deficiency and micronutrient
Obstetric and Newborn Care deficiency.
(BEMONC) facility? A. Epidemiologic transition
A. Performance of assisted deliveries B. Iceberg phenomenon
in imminent breech C. Wasting
B. Newborn resuscitation D. Global malnutrition
C. Manual removal of retained E. Double burden
placenta
236. Refers to the observed trend that the B. Food Fortification
majority of children have mild C. Nutrition Information,
malnutrition, far fewer have moderate Communication and Education
malnutrition and only few have severe D. Micronutrient Supplementation
malnutrition. E. Food assistance
A. Epidemiologic transition
B. Iceberg phenomenon 241. A national project that involves the
C. Wasting voluntary fortification of processed
D. Global malnutrition foods.
E. Double burden A. Medium-Term Philippine Plan of
Action for Nutrition (MTPPAN)
237. Results from inappropriate height-for- B. Sangkap Pinoy Seal
age. C. Maternal and Child Health
A. Wasting D. Early Childhood Development
B. Edema project
C. Emaciation E. Safe Motherhood
D. Stunting
E. None of the above 242. Involves center-based
complementary feeding for wasted
238. Results from inappropriate weight- and stunted pregnant women with
for-height record of delivering low birthweight
A. Wasting infants, and for young children aged 6-
B. Edema 72 months old.
C. Emaciation A. Home, School and Community
D. Stunting Food Production
E. None of the above B. Food Fortification
C. Nutrition Information,
239. It is the country’s first blueprint for Communication and Education
achieving nutritional adequacy for all D. Micronutrient Supplementation
Filipinos which is coordinated by the E. Food assistance
National Nutrition Council.
A. Medium-Term Philippine Plan of 243. Refers to the promotion and
Action for Nutrition (MTPPAN) maintenance of the highest degree of
B. Sangkap Pinoy Seal physical, mental and social well-being
C. Philippine Plan of Action for of workers in all occupations by
Nutrition (PPAN) preventing departures from health,
D. Early Childhood Development controlling risks and the adaptation of
project work to people, and people for their
E. Safe Motherhood jobs.
A. Ergonomics
240. Refers to the addition of a nutrient to B. Occupational health
a food vehicle commonly consumed C. Occupational hazards
by the population to prevent or correct D. Safety
micronutrient deficiencies. E. Occupations
A. Home, School and Community
Food Production
244. Refers to anything that has the A. Chemical hazards
potential to cause harm. B. Ergonomic hazards
A. Accident C. Biologic hazards
B. Risk D. Physical hazards
C. Danger E. Sociologic hazards
D. Hazard
E. Safety 250. Refers to the science of fitting the job
to the worker.
245. Refers to likelihood that a particular A. Ergonomics
hazard will occur. B. Occupational health
A. Accident C. Occupational hazards
B. Risk D. Safety
C. Danger E. Occupations
D. Hazard
E. Safety 251. Which of the following drugs cannot
be used to treat cases of multidrug-
246. Results from mismanagement or resistant (MDR) tuberculosis?
misuse of chemicals resulting in an A. Isoniazid and Pyrazinamide
unacceptable risk to human health. B. Rifampicin and Ethambutol
A. Chemical hazards C. Isoniazid and Rifampicin
B. Ergonomic hazards D. Ethambutol and Pyrazinamide
C. Biologic hazards E. Rifampicin and Pyrazinamide
D. Physical hazards
E. Sociologic hazards 252. Extensively drug-resistant (XDR)
tuberculosis is defined as:
247. Refers to those that result from living A. Resistance to both the first line
in a society where one experiences drugs
noise, lack of privacy and B. Resistance to both first line drugs
overcrowding. and at least one of the six main
A. Chemical hazards classes of second-line drugs
B. Ergonomic hazards C. Resistance to both the first line
C. Biologic hazards drugs and at least three of the six
D. Physical hazards main classes of second-line drugs
E. Sociologic hazards D. Resistance to both first line drugs
and at least four of the six main
248. Include airborne particles, humidity, classes of second-line drugs
equipment design and radiation. E. Resistance to both the first line
A. Chemical hazards and all of the six main classes of
B. Ergonomic hazards second-line drugs
C. Biologic hazards
D. Physical hazards 253. Which of the following is an
E. Sociologic hazards intracellular parasite?
A. Rickettsia
249. Refers to those arising from microbial B. Mycobacterium
decomposition of various substrates C. Bacillus
associated with particular occupations. D. Staphylococcus
E. Streptococcus 259. Malaria infection is initiated by the
bite of a mosquito, which injects this
254. This refers to ringworm infection of form of the protozoan into the
the scalp. bloodstream.
A. Tinea capitis A. Sporozoite
B. Tinea cruris B. Merozoite
C. Tinea pedis C. Trophozoite
D. Tinea unguium D. Gametocyte
E. Tinea versicolor E. Promastigote
260. Hepatitis C virus belongs to this
255. This type of ringworm infection is family
also referred to as onychomycosis. A. Picornaviridae
A. Tinea capitis B. Hepadnaviridae
B. Tinea cruris C. Flaviviridae
C. Tinea pedis D. Deltaviridae
D. Tinea unguium E. Caliciviridae
E. Tinea versicolor
261. This type of hepatitis virus contains a
256. This disease is caused by the virus single strand of RNA, which is not
human parvovirus B19 and results in capable of causing an infection. It
symptoms similar to a mild case of becomes infectious when an external
influenza and a “slapped cheek” facial envelope of HBsAG covers the protein
rash. core of the virus.
A. Exanthema subitum A. HAV
B. Roseola infantum B. HBV
C. Erythema multiforme C. HCV
D. Erythema infectiosum D. HDV
E. Shingles E. HEV

257. This is an example of a 262. The following are vertically-


subcutaneous mycosis. transmitted infections,except:
A. Tinea infection A. Toxoplasmosis
B. Rubella B. Hepatitis B
C. Sporotrichosis C. Herpes simplex
D. Cryptococcosis D. Rubeola
E. Candidiasis E. Cytomegalovirus

258. The causative agent of plague, which 263. This is a measure of the biologically
was known in the middle ages as the degradable organic matter in water
Black Death. A. BOD
A. Sarcoptes scabiei B. BER
B. Yersinia pestis C. BID
C. Francisella tularensis D. GIR
D. Trypanosoma cruzi E. GOD
E. Bacillus anthracis
264. This is a protein product separated 269. This is probably the only bacterium
from the whey by the action of rennin that grows in the peripheral nervous
enzyme in certain bacteria. system.
A. Cheese A. Clostridium tetani
B. Curd B. Clostridium botulinum
C. Butter C. Mycobacterium leprae
D. Cream D. Listeria monocytogenes
E. Casein E. Neisseria meningitidis

265. This is made by churning cream until 270. Strongyloides stercoralis is an


the fatty globules separate. example of:
A. Cheese A. Intestinal nematode
B. Curd B. Extra intestinal nematode
C. Butter C. Trematode
D. Cream D. Anthropods
E. Whey E. NOTA

266. Cell membrane damage causes 271. The following Plasmodium spp.
death because. cause/s benign tertian malaria.
A. The cell undergoes osmotic lysis A. Plasmodium falciparum
B. Cell contents leak out B. Plasmodium vivax
C. The cell plasmolyzes C. Plasmodium ovale
D. The cell lacks a wall D. Plasmodium malariae
E. None of the above E. Both B and C

267. Chlorampenicol binds to the 50s 272. The following Plasmodium spp.
portion of a ribosome, which will cause/s malignant tertian malaria.
interfere with: A. Plasmodium falciparum
A. Transcription in prokaryotic cells B. Plasmodium vivax
B. Transcription in eukaryotic cells C. Plasmodium ovale
C. Translation in prokaryotic cells D. Plasmodium malariae
D. Translation in eukaryotic cells E. Both B and C
E. DNA synthesis
273. Treatment of choice for American
268. Which of the following factors does trypanosomiasis or Chaga’s disease:
not contribute to antibiotic resistance? A. Nifurtimox and Benzidazole
A. Destruction of the plasma B. Pyrimethamine and Sulfadiazine
membrane C. Trimethoprim and
B. Enzymatic destruction or Sulfamethoxazole
inactivation D. Metronidazole
C. Prevention of penetration to the E. Iodoquinol
target site
D. Alteration of the target site 274. Vector of Paragonimiasis.
E. Rapid efflux of the antibiotic A. Mosquito
B. Fly
C. Rat
D. Crab A. Definitive host
E. Snail B. Intermediate host
C. Final host
275. The drug of choice for Bancroftian D. Paratenic host
filariasis. E. Both A and C
A. Pyrimethamine
B. Diethycarbamazine citrate 281. A type of host in which the parasite
C. Mebendazole does not develop further to later
D. Ivermectin stages. However, the parasite remains
E. Praziquantel alive and is able to infect another
susceptible host.
276. The smallest tapeworm infecting A. Definitive host
humans. B. Intermediate host
A. Taenia saginata C. Final host
B. Diphyllobothrium latum D. Paratenic host
C. Hymenolepsis nana E. Both A and C
D. Hymenolepsis diminuta
E. Dipylidium caninum 282. A type of host which harbors the
asexual or larval stage of the parasite.
277. Fasciola hepatica and F. gigantica A. Definitive host
are what type of flukes? B. Intermediate host
A. Lung flukes C. Final host
B. Liver flukes D. Paratenic host
C. Blood flukes E. Both A and C
D. Heterophyid flukes
E. Intestinal flukes 283. This is defined as a permanent
reduction to zero of the worldwide
278. Vector of Hymenolepiasis. incidence of infection caused by a
A. Chigger mite specific agent, as a result of deliberate
B. Tick efforts.
C. Flea A. Disease elimination
D. Fly B. Disease eradication
E. Crab C. Disease prevention
D. Disease reduction
279. A symbiotic relationship where one E. NOTA
organism lives in or on another,
depending on the latter for its survival 284. This refers to the number (usually
and usually at the expense of the host. expressed as percentage) of
A. Commensalism individuals in a population estimated
B. Mutualism to be infected with a particular parasite
C. Parasitism species at a given time.
D. Symbiosis A. Prevalence
E. NOTA B. Incidence
C. Cumulative prevalence
280. A type of host wherein the parasite D. Morbidity
attains sexual maturity. E. NOTA
E. Fasciolopsis buski
285. The drug of choice for the treatment
of trichuriasis. 290. This infection requires Oncomelania
A. Albendazole snails as intermediate hosts.
B. Praziquantel A. Paragonimus westrmani
C. Mebendazole B. Fasciola hepatica
D. Metronidazole C. Schistosoma japonicum
E. Both A and B D. Clonorchis sinensis
E. Fasciolopsis buski
286. Human may serve as both a
definitive host and an intermediate 291. This is an example of a parasite of
host of this type of cestode, therefore the bile duct and the gallbladder of
both intestinal and tissue infections humans and fish-eating mammals.
occur in man. A. Paragonimus westrmani
A. Taenia saginata B. Fasciola hepatica
B. Taenia solium C. Schistosoma japonicum
C. Dipylidium caninum D. Clonorchis sinensis
D. Hymenolepsis nana E. Fasciolopsis buski
E. Hymenolepsis diminuta
292. The following are parasites isolated
287. This parasite is the only human in the placenta, except:
tapeworm, which can complete its A. Toxoplasma
entire life cycle in a single host, B. Trypanosoma
indicating that it does not require an C. Malaria
obligatory intermediate host. D. Trichinella
A. Taenia saginata E. NOTA
B. Taenia solium
C. Dipylidium caninum 293. A diagnostic technique used to
D. Hymenolepsis nana recover eggs of Enterobius
E. Hymenolepsis diminuta vermicularis and Taenia spp.
A. Scotch tape method
288. Also known as Oriental blood fluke. B. Kato katz method
A. Paragonimus westermanii C. Perianal swab
B. Fasciola hepatica D. Direct fecal smear
C. Schistosoma japonicum E. Kato tick method
D. Clonorchis sinensis
E. Fasciolopsis buski 294. A routine method of stool
examination primarily useful in the
289. The causative agent of lung fluke detection of motile protozoan
disease or pulmonary distomiasism or trophozoites using 0.85% NSS.
endemic hemoptysis or parasitic A. Scotch tape method
hemoptysis. B. Kato katz method
A. Paragonimus westrmani C. Perianal swab
B. Fasciola hepatica D. Direct fecal smear
C. Schistosoma japonicum E. Kato tick method
D. Clonorchis sinensis
295. Technique used in detecting eggs D. Wuchereria bancrofti
with tick shells such as Ascaris and E. Brugia malayi
Trichuris, using a mixture of glycerine
and malachite green solution.
A. Scotch tape method
B. Kato katz method
C. Perianal swab
D. Direct fecal smear
E. Kato tick method

296. Common term for Enterobius


vermicularis.
A. Threadworm
B. Fluke
C. Pinworm
D. Flatworm
E. Hookworm

297. Which of the following is also known


as the cat hookworm?
A. Ancylostoma ceylanicum
B. Ancylostoma braziliense
C. Ancylostoma caninum
D. Ancylostoma duodenale
E. Necator americanus

298. Etiologic agent of Chaga’s disease.


A. Trypanosoma cruzi
B. Trypanosoma brucei gambiense
C. Trypanosoma brucei rhodesiense
D. Wuchereria bancrofti
E. Brugia malayi

299. Which of the following is also known


as the Old world hookworm?
A. Trypanosoma cruzi
B. Ancylostoma duodenale
C. Necator americanus
D. Wuchereria bancrofti
E. Brugia malayi

300. Which of the following is also known


as the New world hookworm?
A. Trypanosoma cruzi
B. Ancylostoma duodenale
C. Necator americanus

You might also like